*NURSING > QUESTIONS & ANSWERS > Miami Dade College: 403 REVIEW FINAL EXAM NURS 403/2020 PRIMERA PARTE. Questions and Answers. Graded (All)

Miami Dade College: 403 REVIEW FINAL EXAM NURS 403/2020 PRIMERA PARTE. Questions and Answers. Graded A

Document Content and Description Below

NURS 403/2020 1.A community health nurse in a pediatric clinic is reviewing the history of a 12-year-old client. Which of the following immunizations should the nurse expect to administer? A. Menin... gococcal conjugate B. Herpes Zoster C. Rotavirus D. Pneumococcal polysaccharide 2. Nurse is preparing a community health program for adults at risk for cardiovascular disease. Which of the following should the nurse include as a modifiable risk factor? A. Diagnosis of diabetes Miletus B. Family history of cardiac disease C. Increase age D. Cigarette smoking 3. An occupational health nurse in the clinic of an industrial plant is developing a guidebook for clinic workers. Which of the following actions should the nurse include as a secondary prevention strategy? A. Teach plant workers about proper lifting techniques B. Set up an influenza immunization campaign C. Help plant workers identify signs of carpal tunnel syndrome D. Collaborate with physical therapists to develop program for injured employees to return to work 4. A school nurse receives a call that some children and teachers report being exposed to an undetermined noxious gas odor presenting in the classrooms and are experiencing dizziness. Which of the following actions should the nurse take? A. Have students evacuated from the school and establish a triage area in the school parking lot. B. Move individual who are reporting symptoms to one of the affected classrooms and create a triage area inside C. Arrange for children transportation to the nearest emergency department and the group triage will occur there. D. Transport all children and school personnel to the nearest medical facility school busses 5. The nurse is participating in a biological disaster simulation where citizens are exposed to pneumonic plague. Which of the following interventions should the nurse plan to use while caring for these clients? A. Initiate airborne precaution B. Administer an antitoxin C. Initiate droplet precautions D. Destroy the lines after use6. A nurse is among the first responders to a mass-casualty incident and does not know what type of personal protection equipment (PPE) is needed. Which of the following actions should the nurse take? A. Wait until the type of equipment needed is known B. Decontaminate victims before intervening C. Choose the highest level of protection equipment available. D. Use a dosimeter to measure the level of radiation in the area before intervening 7. A charge nurse is making a room assignment for a client who has scabies. In which of the following rooms should the nurse place the client? A. A negative pressure isolation room B. A semiprivate room with a client who has pediculosis capitis C. A positive pressure isolation room D. A private room. 8. A nurse is performing triage for a group of clients following a mass casualty incident (MCI). Which of the following clients should the nurse plan to care for first? A. A client experiencing a tension pneumothorax B. A client who has a closed upper extremity fracture C. A client who has full thickness burns over 80% of this body D. A client who has agonal respirations 9. a nurse is helping to triage a group of clients at a mass casualty incident who were involved in an explosion at a local factory. Which of the following clients should the nurse tag to be the priority for care? A. A client who has severe head injury respiratory rate 6/min and unresponsive B. A client who has a simple fracture of the femur, multiple scratches on both legs, and crying hysterically C. A client who has a piece of wood punctured into the chest wall and has an audible hissing sound coming from the wound site. D. A female who is pregnant at 20 weeks of gestation has multiple cuts and abrasions and is walking around 10. A nurse is triaging victims of a multiple motor-vehicle crash. The nurse assesses a client trapped under a car who is apneic and has a weak pulse at 120/min. After repositioning his upper airway, the client remains apneic. Which of the following actions should the nurse take? A. Start CPR B. Place a red tag on the client’s upper body and attempt to help the next client in need C. Place a black tag on the client’s upper body and attempt to help the next client in need. D. Reposition the client upper airway a second time before assessing his respirations 11. A nurse is reviewing the guidelines for reporting nationally notifiable infectious diseases. Which of the following diseases should the nurse plan to report to the Centers for Disease Control and Prevention (CDC)? A. Lyme disease B. Cytomegalovirus C. Streptococcus pharyngitis D. Toxoplasmosis12. A community health nurse is reviewing the levels of disease prevention. Which of the following activities is an example of tertiary prevention? A. Providing treatment for clients who have chronic obstructive pulmonary disease. B. Performing screening for sexually transmitted infections C. Administering influenza immunizations at local health fair D. Testing new nurses for exposure to TB 13. A charge nurse is discussing staff nurses’ responsibilities in preplanning for response to a disaster. Which of the following responsibilities should the nurse include in the discussion? A. Identify community resources that are available. B. Evaluate the impact of a disaster on the community C. Asses survivors of a disaster for level of psychological stress D. Link victims with support agencies to help with food clothing shelter and counseling needs 14. A nurse is reviewing treatment protocols for clients exposed to bioterrorism agents. For which of the following agents should the nurse plan to administer a vaccine following exposure? A. Anthrax B. Botulism C. Plague D. Smallpox 15. A nurse is providing staff education about smallpox as a bioterrorism threat. Which of the following statements indicates an understanding of this agent? (Select all that apply) A. Smallpox is transmitted person to person B. Infection is characterized by severe respiratory distress C. Smallpox vaccination ensures lifelog immunity D. Naturally occurring smallpox has been eradicated from the world. E. Smallpox is often confused with varicella. 16. A nurse is preparing to care for a client who has suspected exposure to plague as a result of bites from infected fleas. Which of the following actions should the nurse plan to take first? A. Prepare for drainage of lesions B. Initiate contact precautions. C. Notify the CDC control and preventions D. Decontaminate the client 17. A nurse is reinforcing teaching with a group of adolescent females who are pregnant about expected changes related to pregnancy. Which of the following client statements indicates understanding of the teaching? A. It is normal to have a white vaginal discharge. B. I should recognize fetal movement by 12 weeks C. I will take fluid pills if my ankles begin to swell D. My nipples and areolae will become pale as my breast enlarge 18. A nurse is preparing to administer ciprofloxacin to a client. The nurse identify that the medication is treated for exposure to which of the following agents? A. Smallpox B. Sarin gas C. Ebola virus D. Antrax19. A nurse is caring for a client who suspects recent exposure to inhalation anthrax. Which of the following findings indicate possible exposure? A. Flu-like symptoms. B. Sloughing of skin C. Vesicles on the skin D. Respiratory failure 20. A nurse is reviewing information about the Health Insurance Portability and Accountability Act (HIPAA) with a newly licensed nurse. Which of the following statements by the newly licensed nurse indicates a need for further teaching? A. Information about a client can be disclosed to family members at any time. B. HIPAA established regulations of individually identifiable health information in verbal electronic or written form C. A client address would be an example of personally identifiable information D. HIPAA is a federal law not a state low 21. A nurse is preparing to perform hand hygiene. Which of the following actions should the nurse take? A. Adjust the water temperature to feel hot B. Apply 4 to 5 ml of liquid soap to the hands. C. Hold the hands higher than to the elbows D. Rub hands and arm to dry 22. A nurse is preparing to administer three liquid medications to a client who has an NG feeding tube with continuous enteral feeding. Which of the following actions should the nurse take? A. Mix the three medications together prior to administering B. Dilute each medication with 10 ML of tap water C. Maintain the head of the bed in a flat position for 30 min following medications administration D. Flush the NG feeding tube with 30 ml of water immediately following medication administration. 23.- a nurse is administering nasal decongestant drops for a client. Which of the following actions should the nurse take? A. Tell the client to blow her nose gently before the instillation. B. Assistant the client to a side lying position C. Hold the dropper 2 cm (1in) above the naris D. Instruct the client to say in the same position for 2 min 24.- A nurse is performing tracheostomy care for a client and suctioning to remove copious secretions. Which of the following actions should the nurse take? A. Suction two to three times with a 60-second pause between passes. B. Perform chest physiotherapy prior to suctioning C. Lubricate the suction catheter tip with sterile saline D. Hyperventilate the client on 100% oxygen prior to suctioning 25.- The newly licensed nurse is applying prescribed wrist restraints on a client. Which of the following action should the nurse take? A. Secure the restraints using a quick-release tie. B. Ensure four fingers fit under the restraints to prevent constriction C. Secure the restraints to the lowest bar of the side rail D. Anticipate removing the restraints every 4 hours26.- A nurse is preparing a client for outpatient surgery. After the nurse inserts the IV catheter, the client reports pain in the insertion area. Which of the following actions should the nurse take? A. Remove the catheter and insert another into a different site. B. Administer an analgesic PO C. Request a prescription for placement of a central venous access device D. Administer local anesthetic 27.- A client smoking in his bathroom has dropped a cigarette butt into a wastepaper basket, which begins to smolder. Which of the following actions is the nurse`s priority? A. Close the fire doors on the unit B. Activate the alarm C. Remove any client in the immediate vicinity D. Use the fire extinguisher to put out the fire 28. A nurse is caring for a client who refuses treatment and asks to be discharged from the hospital against medical advice. The nurse notifies the client’s provider, who tells the nurse to restrain the client. If necessary, to keep her from leaving the hospital. The nurse understands that restraining this client would be considered which type of civil action by the nurse? A. Invasion of privacy B. Assault C. Battery D. False imprisonment 29.- A nurse is inserting an IV catheter for an older client in preparation for an outpatient procedure. Which of the following veins should the nurse select? A. Dorsal metacarpal vein B. Radial vein in the wrist C. Antecubital vein D. Median vein in the forearm. 30. A nurse is providing oral care for a client who is immobile. Which of the following actions should the nurse take? A. Use a stiff toothbrush to clean the client teeth B. Use the thumb and index to keep the client mouth open C. Turn the client on his side before starting oral care D. Apply petroleum jelly to the client lips after oral care 31. A nurse is caring for a client who has a prescription for a stool test for guaiac. The nurse understands the purpose of the test is to check the stool for which of the following substances? A. Steatorrhea B. Blood C. Bacteria D. Parasites 32.- A nurse is instructing a client who has a new diagnosis of Raynaud`s disease about preventing the onset of manifestations. Which of the following client statements should indicate to the nurse the need for additional teaching? A. I will wear gloves when removing food from the freezer B. I will try anticipating and avoid stressful situation when possible C. I will complete the smoking cessation program I startedD. I will take my medications at the first sign of an attack. 33.- A nurse is caring for a client who experienced a lacerated spleen and has been on bedrest for several days. The nurse auscultates decreased breath sounds in the lower lobes of both lungs. The nurse should realize that this finding is most likely an indication of which of the following conditions. A. An upper respiratory infection B. Pulmonary edema C. Atelectasis. D. Hypertension and crackles 34. The nurse is assessing a client and discovers the infusion pump with the client’s total parenteral nutrition (TPN) solution is infusing. The nurse should monitor the client for which of the following conditions? A. Excessive thirst and urination B. Shakiness and diaphoresis C. Fever and chills D. Hypertension and crackles 35. A nurse is caring for four clients who have drainage tubes. Which of the following clients should the nurse recognize as being at risk for hypokalemia? A. The client who has a tracheostomy tube attached to humidified oxygen B. The client who has an indwelling urinary catheter to gravity drainage C. The client who has a chest tube to water seal D. The client who has a nasogastric (NG) tube to suction. 36.- A nurse is caring for a client who has an indwelling unitary catheter and notes blood-tinged urine in the catheter bag. The nurse recognizes this finding can be a manifestation of which of the following urinary alterations? A. Pernicious anemia B. Dehydration C. Prostate enlargement D. Bladder infection 37.- A nurse is assessing a client`s cranial nerves as part of a neurological examination. Which of the following actions should the nurse take to assess cranial nerve III? A. Testing visual acuity B. Observing for facial symmetry C. Eliciting the gag reflex D. checking the pupillary response to light. 38.- A nurse is caring for a client who has had an allogeneic hematopoietic stem-cell transplant. Which of the following infection-control precautions should the nurse use while caring for this client? A. Airborne B. Protective. C. Contact D. Droplet 39.- An assistive personnel (AP) reports a client`s vital signs as tympanic Temperature 37.1º C (98.8º F), pulse 92/min, respiratory rate 18/min, and BP 98/58 mm Hg. Which of the following vital signs should the nurse re-measure? A. BPB. Respiratory rate C. Pulse rate D. temperature 40. A nurse accidentally administers the wrong medication to a client, which results in a severe allergic reaction and prolongs the client’s hospitalization. The client could rightfully sue the nurse for which of the following? A. Battery B. Assault C. Malpractice. D. Abuse 41. A nurse is caring for an older adult client who states, “I am afraid that I may fall white walking to the bathroom during the night.” Which of the following actions should the nurse take? A. Limit the client fluid intake in the evening B. Obtain a bedside commode for the client use C. Leave a nightlight on the client’s room. D. Put the side rails up and tell the client to call the nurse before voiding 42.- A nurse is working with a licensed practical nurse (LPN) to care for a client who is receiving a continuous IV infusion. Which of the following findings reported by the LPN indicated to the nurse the client has phlebitis at the IV insertion side? A. The infusion rate has stopped but the tubing is not kinked B. The area surrounding the insertion site feels warm to the touch. C. There is fluid leaking around the insertion site D. There is not blood return when the tubbing is aspirated 43.- A nurse in the emergency department is caring for a client who collapsed after playing football on a hot day. After reviewing the admission laboratory findings, the nurse recognizes are consistent with which of the following conditions? Sodium 152 mEq/L, Potassium 3.6 mEq/L, Chloride 105 mEq/L, Glucose 102 mg/dL BUN 18 mg/dL, Creatinine 0.7mg/dL A. Renal failure B. Low protein C. Dehydration. D. SIADH 44.- A nurse at an extended- care facility is instructing a class assistive personnel (AP) about client use of assistive devises during ambulation. Which of the following instruction should the nurse give the AP about the clients` use of a cane? A. When the client moves, he should move the cane forward first. B. The client should hold the cane on the strong side of his body C. The grip should be level with the client waist D. The client should first move the strong leg then the weak one 45. The nurse is caring for a client who has active pulmonary tuberculosis (TB). The client requires airborne precautions and is receiving multidrug therapy. Which of the following precautions should the nurse take to transport the client safety to the radiology department for a chest x-ray? A. Ask the X ray technician to come to the client’s room to obtain a portable x ray B. Have the client wear a mask C. Notify the x ray department that the client requires airborne precautions D. Wear a filtration mask and gloves during transport46. A nurse is assisting with transferring a client from the bed to wheelchair. Which of the following actions should the nurse take? A. Place the wheelchair at 90 grade angles to the bed B. Lock the wheels of the bed and the wheelchair. C. Acquire the help of several people to lift the client D. Elevate the bed to a position of comfort to the nurse MIDTER PIC FROM AM National Prevention Strategy targets intervention in multiple setting to move the health of the populations from A. Poor to rich B. Wellness to health C. Shift from sickness to prevention D. Sadness to happiness Rational: Called for under the Patient Protection and Affordable Care Act Developed by National Prevention Council Four Strategic Directions: 1. Building healthy and safe community environments 2. Expanding quality preventive services in both clinical and community settings 3. Empowering people to make healthy choices 4. Eliminating health disparities To understand the causes of health and disease, epidemiology studies: a. Individuals b. Families c. Groups d. Populations a nurse is teaching a client diagnosed with gonorrhea how to prevent reinfection and further spread. Which of the following describes the action taken by the nurse? Primary prevention Secondary promotion Tertiary prevention Secondary prevention Secondary prevention focuses on early detection and prompt treatment of disease, injury, or disability. Primary prevention refers to those interventions aimed at focuses on chronic care and rehabilitation, Primary health care refers to the first line of care provided to patients typically by a physician or other health care provider. PHNs utilize registries to identify children with delayed or missing immunizations. They subsequently follow up with families by phone calls or home visits. Which of the following levels of practice is being implemented? Policy Systems Individual Community A nurse who is involved in identifying individuals with unrecognized health risk factors or asymptomatic disease is using which intervention? a. Screening b. Referral and follow-up c. Surveillanced. Health teaching The factors, exposures, characteristics, and behaviors that determine patterns of disease are described using: a. descriptive epidemiology. b. analytic epidemiology. c. distribution. d. determinants. Determinants are the factors, exposures, characteristics, and behaviors that determine patterns of disease, which may be individual, relational, social, communal, or environmental. Descriptive epidemiology seeks to describe the occurrence of a disease in terms of person, place, and time. Analytic epidemiology focuses on the investigation of causes and associations. Distribution describes who has the disease and where and when the disease occurs. A client is receiving PHC services. Which of the following expectations would the client have? a. The care would be provided by a physician. b. Private insurance would be necessary to receive care. c. The emphasis of care provided would be on prevention. d. A referral would be made to a specialist after receiving services. The focus of PHC is a broad range of services that are designed to be affordable for the recipients of the care and the governments who provide them. The emphasis of care is on prevention and the means of providing the care is based on practical, scientifically sound, culturally ap-propriate, and socially acceptable methods. All clients can receive PHC services regardless of insurance status. Those receiving primary care services may or may not need referral to a specialist. which of the following is an example of public health agency. Federal Bureau of Investigation (FBI) Drug enforcement administration (DEA) Women, Infants and children (WIC) Medical Center clinics (MCC) the public health care functions are directly linked to the healthcare promotion and prevention levels. The tertiary levels is characterized by The services are designed to treat disease in acute stage The service are designed to limit the progression of the disease or disability The function is to detect and treat disease is early stage Meets the basic needs of health people A school nurse is teaching a group of nurses newly hired to work in the school system about pedi information is appropriate to include in the teaching? Nits that are shed into the environment are capable of hatching for up to 10 days Infestation often appears as grayish brown threadlike burrow along the skin behind the ears Family pets can contribute to the transmission of lice Children from lower socioeconomic areas are more likely to have lice. 1.A community health nurse in a pediatric clinic is reviewing the history of a 12-year-old client. Which of the following immunizations should the nurse expect to administer? a.Meningococcal conjugate 2. Nurse is preparing a community health program for adults at risk for cardiovascular disease. Which of the following should the nurse include as a modifiable risk factor? d. Cigarette smoking 3. An occupational health nurse in the clinic of an industrial plant is developing a guidebook for clinic workers. Which of the following actions should the nurse include as a secondary prevention strategy?c. Help plant workers identify signs of carpal tunnel syndrome 4. A school nurse receives a call that some children and teachers report being exposed to an undetermined noxious gas odor presenting in the classrooms and are experiencing dizziness. Which of the following actions should the nurse take? a. Have students evacuated from the school and establish a triage area in the school parking lot. 5. The nurse is participating in a biological disaster simulation where citizens are exposed to pneumonic plague. Which of the following interventions should the nurse plan to use while caring for these clients? c. Initiate droplet precautions 6. A nurse is among the first responders to a mass-casualty incident and does not know what type of personal protection equipment (PPE) is needed. Which of the following actions should the nurse take? c. Choose the highest level of protection equipment available. 7. A charge nurse is making a room assignment for a client who has scabies. In which of the following rooms should the nurse place the client? d. A private room. 8. A nurse is performing triage for a group of clients following a mass casualty incident (MCI). Which of the following clients should the nurse plan to care for first? a. A client experiencing a tension pneumothorax 9. a nurse is helping to triage a group of clients at a mass casualty incident who were involved in an explosion at a local factory. Which of the following clients should the nurse tag to be the priority for care? c. A client who has a piece of wood punctured into the chest wall and has an audible hissing sound coming from the wound site. 10. A nurse is triaging victims of a multiple motor-vehicle crash. The nurse assesses a client trapped under a car who is apneic and has a weak pulse at 120/min. After repositioning his upper airway, the client remains apneic. Which of the following actions should the nurse take? c. Place a black tag on the client’s upper body and attempt to help the next client in need. 11. A nurse is reviewing the guidelines for reporting nationally notifiable infectious diseases. Which of the following diseases should the nurse plan to report to the Centers for Disease Control and Prevention (CDC)? a. Lyme disease 12. A community health nurse is reviewing the levels of disease prevention. Which of the following activities is an example of tertiary prevention? a. Providing treatment for clients who have chronic obstructive pulmonary disease. 13. A charge nurse is discussing staff nurses’ responsibilities in preplanning for response to a disaster. Which of the following responsibilities should the nurse include in the discussion?a. Identify community resources that are available. 14. A nurse is reviewing treatment protocols for clients exposed to bioterrorism agents. For which of the following agents should the nurse plan to administer a vaccine following exposure? d. Smallpox 15. A nurse is providing staff education about smallpox as a bioterrorism threat. Which of the following statements indicates an understanding of this agent? (Select all that apply) a. Smallpox is transmitted person to person d. Naturally occurring smallpox has been eradicated from the world. E. Smallpox is often confused with varicella. 16. A nurse is preparing to care for a client who has suspected exposure to plague as a result of bites from infected fleas. Which of the following actions should the nurse plan to take first? b. Initiate contact precautions. 17. A nurse is reinforcing teaching with a group of adolescent females who are pregnant about expected changes related to pregnancy. Which of the following client statements indicates understanding of the teaching? a. It is normal to have a white vaginal discharge. 18. A nurse is preparing to administer ciprofloxacin to a client. The nurse identify that the medication is treated for exposure to which of the following agents? d. Antrax 19. A nurse is caring for a client who suspects recent exposure to inhalation anthrax. Which of the following findings indicate possible exposure? a. Flu-like symptoms. 20. A nurse is reviewing information about the Health Insurance Portability and Accountability Act (HIPAA) with a newly licensed nurse. Which of the following statements by the newly licensed nurse indicates a need for further teaching? a. Information about a client can be disclosed to family members at any time. 21. A nurse is preparing to perform hand hygiene. Which of the following actions should the nurse take? B.- Apply 4 to 5 ml of liquid soap to the hands. 22. A nurse is preparing to administer three liquid medications to a client who has an NG feeding tube with continuous enteral feeding. Which of the following actions should the nurse take? D.- Flush the NG feeding tube with 30 ml of water immediately following medication administration. 23.- a nurse is administering nasal decongestant drops for a client. Which of the following actions should the nurse take? A.- Tell the client to blow her nose gently before the instillation.24.- A nurse is performing tracheostomy care for a client and suctioning to remove copious secretions. Which of the following actions should the nurse take? A.- Suction two to three times with a 60-second pause between passes. 25.- The newly licensed nurse is applying prescribed wrist restraints on a client. Which of the following action should the nurse take? a. Secure the restraints using a quick-release tie. 26.- A nurse is preparing a client for outpatient surgery. After the nurse inserts the IV catheter, the client reports pain in the insertion area. Which of the following actions should the nurse take? B.- Remove the catheter and insert another into a different site. 27.- A client smoking in his bathroom has dropped a cigarette butt into a wastepaper basket, which begins to smolder. Which of the following actions is the nurse`s priority? a.Move any clients in the immediate vicinity 28. A nurse is caring for a client who refuses treatment and asks to be discharged from the hospital against medical advice. The nurse notifies the client’s provider, who tells the nurse to restrain the client. If necessary, to keep her from leaving the hospital. The nurse understands that restraining this client would be considered which type of civil action by the nurse? d. False imprisonment 29.- A nurse is inserting an IV catheter for an older client in preparation for an outpatient procedure. Which of the following veins should the nurse select? d.- Median vein in the forearm. 30. A nurse is providing oral care for a client who is immobile. Which of the following actions should the nurse take? c. Turn the client on his side before starting oral care. 31. A nurse is caring for a client who has a prescription for a stool test for guaiac. The nurse understands the purpose of the test is to check the stool for which of the following substances? b. Blood 32.- A nurse is instructing a client who has a new diagnosis of Raynaud`s disease about preventing the onset of manifestations. Which of the following client statements should indicate to the nurse the need for additional teaching? D.- I will take my medications at the first sign of an attack. 33.- A nurse is caring for a client who experienced a lacerated spleen and has been on bedrest for several days. The nurse auscultates decreased breath sounds in the lower lobes of both lungs. The nurse should realice that this finding is most likely an indication of which of the following conditions? C.- Atelectasis. 34. The nurse is assessing a client and discovers the infusion pump with the client’s total parenteral nutrition (TPN) solution is infusing. The nurse should monitor the client for which of the following conditions?b. Shakiness and diaphoresis 35. A nurse is caring for four clients who have drainage tubes. Which of the following clients should the nurse recognize as being at risk for hypokalemia? d. The client who has a nasogastric (NG) tube to suction. 36.- A nurse is caring for a client who has an indwelling unitary catheter and notes blood-tinged urine in the catheter bag. The nurse recognizes this finding can be a manifestation of which of the following urinary alterations? D.- Bladder infection 37.- A nurse is assessing a client`s cranial nerves as part of a neurological examination. Which of the following actions should the nurse take to assess cranial nerve III? D.- checking the pupillary response to light. 38.- A nurse is caring for a client who has had an allogeneic hematopoietic stem-cell transplant. Which of the following infection-control precautions should the nurse use while caring for this client? B.- Protective. 39.- An assistive personnel (AP) reports a client`s vital signs as tympanic Temperature 37.1º C (98.8º F), pulse 92/min, respiratory rate 18/min, and BP 98/58 mm Hg. Which of the following vital signs should the nurse re-measure? A.- BP 40. A nurse accidentally administers the wrong medication to a client, which results in a severe allergic reaction and prolongs the client’s hospitalization. The client could rightfully sue the nurse for which of the following? c. Malpractice. 41. A nurse is caring for an older adult client who states, “I am afraid that I may fall white walking to the bathroom during the night.” Which of the following actions should the nurse take? c. Leave a nightlight on the client’s room. 42.- A nurse is working with a licensed practical nurse (LPN) to care for a client who is receiving a continuous IV infusion. Which of the following findings reported by the LPN indicated to the nurse the client has phlebitis at the IV insertion side? B.- The area surrounding the insertion site feels warm to the touch. 43.- A nurse in the emergency department is caring for a client who collapsed after playing football on a hot day. After reviewing the admission laboratory findings, the nurse recognizes are consistent with which of the following conditions? Sodium 152 mEq/L Potassium 3.6 mEq/L Chloride 105 mEq/L Glucose 102 mg/dLBUN 18 mg/dL Creatinine 0.7mg/dL C.- Dehydration. 44.- A nurse at an extended- care facility is instructing a class assistive personnel (AP) about client use of assistive devises during ambulation. Which of the following instruction should the nurse give the AP about the clients` use of a cane? A.- When the client moves, he should move the cane forward first. 45. The nurse is caring for a client who has active pulmonary tuberculosis (TB). The client requires airborne precautions and is receiving multidrug therapy. Which of the following precautions should the nurse take to transport the client safety to the radiology department for a chest x-ray? b. Have the client wear a mask 46. A nurse is assisting with transferring a client from the bed to wheelchair. Which of the following actions should the nurse take? b. Lock the wheels of the bed and the wheelchair. 47. Prueba de Mantoux positiva R/ Poner al paciente con presión negativa. 48. Paciente alterado R/ Poner al paciente cerca de la estación de enfermería 49. Paciente que se atora. R/ HEIMLICH MANEUVER 50. Qué paciente se pone cerca de la estación de enfermería. R/ Paciente con lesión cerebral. NURS 403/2020 SEGUNDA PARTE. 1. A nurse is establishing health promotion goals for a female client who smokes cigarettes, has htn, and has a bmi of 26. which of the following goals should the nurse include? the client will walk for 30 min 5 days a week 2. A public health nurse is assessing an older adult client who lives with a family member. the nurse identifies several bruises in various stages of healing. the client and family member explain that the bruises are a result of clumsiness. however, based on the distribution of the bruises, the nurse suspects abuse. which of the following actions should the nurse take first? report the findings to a supervisor 3. A community health nurse is developing a pamphlet about breast self-examination (BSE) for a local health fair. which of the following instructions should the nurse include? breasts can be examined in the shower with soapy hands4. A community health nurse in a pediatric clinic is reviewing the history of 12 yo client. which of the following immunizations should the nurse expect to administer? meningococcal conjugate 5. A community health nurse is conducting an educatioal program on various environmental pollutants. the nurse should emphasize that clients who have which of the following disorderes are especially vulnerable to ozone effects? asthma 6. A nurse working for a home health agency is assessing an older adult male client. which of the following findings is the priority for the nurse to address? dysphagia 7. A nurse is preparing a community health program for adults at risk for cv dz. which of the following should the nurse include as a modifiable risk factor? cigarette smoking 8. A school nurse is teaching a group of nurses newly hired to work in the school system about pediculosis captitis (head lice). which of the following information is appropriate to include in the learning? nits that are shed into the environment are capable of hatching for up to 10 days 9. An older adult client who lives alone tells a clinic nurse that he is unable to drive himself to the store and is afraid to cook on the stove. which of the following community resources should the nurse recommend for this client? meals on wheels 10. A nurse is working with an interdisciplinary disaster committee to develop a communitywide emergency response plan in the event of a nonbiological or chemical incident. the nurse should include which of the following agencies to be notified immediately after calling 911? A. Office of Emergency Management (OEM) B. Federal Emergency Management Agency (FEMA) C. American Red Cross (ARC) D. U.S. Department of Homeland Security (DHS) 11. An occupational health nurse in the clinic of an industrial plant is developing a guidebook for clinic workers. which of the following actions should the nurse include as a secondary prevention strategy? help plant workers identify signs of carpal tunnel syndrome 12. A public health nurse is teaching a group of nurses about smallpox. which of the following statements by one of the nurses indicates understanding of the teaching? unlike chickenpox, the vesicles of smallpox are more abundant on the face13. A school nurse receives a call that some children and teachers report being exposed to an undetermined noxious gas odor presenting in the classrooms and are experiencing dizziness. which of the following actions should the nurse take? have students evacuated from the school and establish a triage area in the school parking lot 14. A nurse is working with an emergency response team in caring for a group or people who may have been exposed to anthrax while doing farm work but are not exhibiting manifestation of illness. which of the following is the appropriate action for the nurse to take? A. Place the clients in isolation. B. Initiate client decontamination. C. Administer antibiotic therapy as prescribed. D. Treat clients with an antitoxin. 15. A nurse is participating in a biological disaster simulation where citizens are exposed to pneumonic plague. which of the following isolation precautions should the nurse plan to use while caring for these clients. droplet 16. A nurse is among the first responders to a mass-casualty incident and does not know what type of personal protective equipment (PPE) is needed. which of the following actions should the nurse take? choose the highest level of protection equipment available 17. A charge nurse is making a room assignment for a client who has scabies. in which of the following rooms should the nurse place the client? a private room 18. A nurse is performing triage for a group of clients following a mass casualty incident (MCI). which of the following clients should the nurse plan to care for first? a client experiencing a tension pneumothorax 19. A nurse is helping to triage a group of clients at a mass casualty incident who were involved in an explosion at a local factory. which of the following clients should the nurse tag to be the priority for care? A. A client who has severe head injuries, respiratory rate 6/min, and is unresponsive B. A client who has a simple fracture of the femur, multiple scratches on both legs, and is crying hysterically C. A client who has a piece of wood punctured into the chest wall and has an audible hissing sound coming from the wound site D. A female who is pregnant at 20 weeks of gestation, has multiple cuts and abrasions, and is walking arounda client who has a piece of wood punctured into the chest wall and has an audible hissing sound coming from the wound site 20. A nurse is assisting with field triage following a motor-vehicle crash involving a bus with multiple victims. the nurse assesses a child who has an open fracture of the femur. which of the following actions should the nurse take? A. Locate the child's parents to obtain consent for treatment. B. Place a yellow triage tag on the child. C. Notify the emergency department of the child's imminent arrival. D. Perform a complete head-to-toe assessment. 21. A nurse is triaging victims of a multiple motor-vehicle crash. the nurse assesses a client trapped under a car who is apneic and has a weak pulse at 120/min. after repositioning his upper airway, the client remains apneic. which of the following actions should the nurse take? place a black tag on the client upper body and attempt to help the next client in need 22. A nurse is triaging clients injured during a tornado. the nurse assesses a client who has an open fracture of his arm. which of the following actions should the nurse take? place a yellow tag on the client's upper body 23. A nurse is the triage officer in the emergency department when 4 clients arrive following a factory explosion. which of the following clients should the nurse care for first? an unconscious adult client who has a sucking chest wound, respirations of 38/min, and capillary refill of <2 seconds 24. A nurse is education community members about how to prepare for a disaster. which of the following supplies should the nurse instruct the clients to include in a disaster preparedness kit? SATA clean clothing, personal identification, matches, prescription meds 25. A nurse is assisting with triaging clients following an explosion. which of the following clients should the nurse identify as the highest priority? a conscious adult client who has second-degree burns on both lower legs; resp rate is >30/mi 26. A nurse is reviewing the guidelines for reporting nationally notifiable infectious diseases. which of the following diseases should the nurse plan to report to the centers for disease control and prevention (CDC)? lyme disease 27. A community health nurse is reviewing the levels of disease prevention. which of the following activities is an example of tertiary prevention?providing treatment for clients who have COPD 28. A charge nurse is discussing staff nurses' responsibilities in preplanning for response to a disaster. which of the following responsibilities should the nurse include in the discussion? identify community resources that are available 29. A nurse is reviewing treatment protocols for clients exposed to bioterrorism agents. for which of the following agents should the nurse plan to administer a vaccine following exposure? smallpox 30. A nurse is providing staff education about smallpox as a bioterrorism threat. which of the following statements indicates an understanding of this agent? SATA smallpox is transmitted person to person naturally occurring smallpox has been eradicated from the world smallpox is often confused with varicella 31. A nurse is preparing a response protocol for botulism as a bioterrorism agent. the nurse should prepare the protocol based on which of the following information? SATA botulism can produce paralysis within 12 to 72 hrs following exposure vomiting and diarrhea are expected findings follwoing exposure 32. A nurse is preparing to care for a client who has suspected exposure to plague as a result of bioterrorism. which of the following actions should the nurse plan to take first? initiate contact precautions 33. A nurse is planning a staff education session regarding biological weapons of mass destruction. which of the following should he plan to include in the session? SATA smallpox anthrax botulism 34. A charge nurse is discussing the phases of community response to disaster with nursing staff. which of the following statements indicates an understanding of the heroic phase of disaster response? A. "Personnel are willing to work in dangerous conditions to provide assistance." (During the heroic phase, responders are willing to work in dangerous conditions for long periods of time to provide assistance.) B. "Survivors come together and share stories of survival." C. "Responders experience exhaustion due to extended relief eorts." D. "Normalcy begins to return to the community." personnel are willing to work in dangerous conditons to provide assistance35. A community health nurse is providing a community education program about disaster preparedness. which of the following should the nurse recommend that clients include in their family's disaster readiness supply kit or 'go bag'? pencil and paper whistle(pito para silvar) copies of insurance cards household bleach 36. A nurse is reinforcing teaching with a group of adolescent females who are pregnant about expected changes related to pregnancy. which of the following client statements indicates understanding of the teaching? it is normal to have a white vaginal discharge 37. A nurse is leading a therapeutic group for clients at an output mental health clinic. which of the following statements indicates a problem with role transition? i just can’t seem to find any energy to take care of my children since my husband divorced me 38. A nurse is preparing to administer cipro to a client. the nurse should identify that the medication is treatment for exposure to which of the following agents anthrax 39. A nurse is caring for a client who suspects recent exposure to inhalation anthrax. which of the following findings indicates possible exposure? flu-like symptoms NURS 403/2020 TERCERA PARTE 1.A nurse is establishing health promotion goals for a female client who smokes cigarettes, has hypertension, and has a BMI of 26. Which of the following goals should the nurse include? The client will walk for 30 min 5 days a week 2. A public health nurse is assessing an older adult client who lives with a family member. The nurse identifies several bruises in various stages of healing. The client and family member explain that the bruises are a result of clumsiness. However, based on the distribution of the bruises, the nurse suspects abuse. Which of the following actions should the nurse take first? Report the findings to a supervisor 3. A community health nurse is developing a pamphlet about breast self-examination (BSE) for a local health fair. Which of the following instruction should the nurse include? Expect some breast dimpling or discharge with age 4. A community health nurse in a pediatric clinic is reviewing the history of a 12 years old client. Which of the following immunizations should the nurse expect to administer? Meningococcal conjugate5. A CHN is conducting an education program on various environmental pollutants. The nurse should emphasize that clients who have which of the following disorders are especially vulnerable to ozone effects? Asthma 6. A nurse working for a home health agency is assessing an older adult male client. Which of the following findings is the priority for the nurse to address? Dysphagia 7. A nurse is preparing a community health program for adults at risk for cardiovascular disease. Which of the following should the nurse include as a modifiable risk factor? Cigarette smoking 8. A school nurse is teaching a group of nurses newly hired to work in the school system about pediculosis capitis (head lice). Which of the following information is appropriate to include in the teaching? Nits that are shed into the environment are capable of hatching for up to 10 days 9. An older adult client who lives alone tells a clinic nurse that he is unable to drive himself to the store and is afraid to cook on the stove. Which of the following community resources should the nurse recommend for this client? Meals on Wheels 10. A nurse is working with an interdisciplinary disaster committee to develop a community-wide emergency response plan in the event of a nonbiological or chemical incident. The nurse should include which of the following agencies to be notified immediately after calling 911? Office of Emergency Management (OEM) 11. An occupational health nurse in the clinic of an industrial plant is developing a guidebook for clinic workers. Which of the following actions should the nurse include as a secondary prevention strategy? Help plant workers identify signs of carpal tunnel syndrome 12. A public health nurse is teaching a group of nurses about smallpox. Which of the following statements by one of the nurses indicates understanding of the teaching? Unlike chickenpox, the vesicles of smallpox are more abundant on the face 13. A school nurse receives a call that some children and teachers report being exposed to an undetermined noxious gas odor presenting the classrooms and are experiencing dizziness. Which of the following actions should the nurse take? A. Have students evacuated from the school and establish a triage area in the school parking lot. B. Move individuals who are reporting symptoms to one of the aected classrooms and create a triage area inside the room. C. Arrange for client transportation to the nearest emergency department and tell the group triage will occur there.D. Transport all children and school personnel to the nearest medical facility using school buses Have students evacuated from the school and establish a triage area in the school parking lot 14.A nurse is working with an emergency response team in caring for a group of people who may have been exposed to anthrax while doing farm work but are not exhibiting manifestation of illness. Which of the following is the appropriate action for the nurse to take? Administer antibiotic therapy 15.A nurse is participating in a biological disaster simulation where citizens are exposed to pneumonic plague. Which of the following isolation precautions should the nurse plant to use while caring for these clients. Initiate Droplet precaution 16.A nurse is among the first responders to a mass-casualty incident and does not know what type of personal protective equipment PPE is needed. which of the following actions should the nurse take? Choose the highest level of PPE available 17.A charge nurse is making a room assignment for a client who has scabies. In which of the following rooms should the nurse place the client? A private room 18.A nurse is performing triage for a group of clients following a mass casualty incident (MCI). Which of the following clients should the nurse plan to care for first? A. A client experiencing a tension pneumothorax B. A client who has a closed upper extremity fracture C. A client who has full-thickness burns over 80% of his body D. A client who has agonal respirations A client experiencing a tension pneumothorax. 19.A nurse is helping to triage a group of clients at a mass casualty incident who were involved in an explosion at a local factory. Which of the following clients should the nurse tag to be the priority for care? A client who has a piece of wood punctured into the chest wall and has an audible hissing sound coming from the wound site 20.A nurse is assisting with field triage following a motor-vehicle crash involving a bus with multiple victims. The nurse assesses a child who has an open fracture of the femur. Which of the following actions should the nurse take? Place a yellow tag on the child 21.A nurse is triaging victims of a multiple motor-vehicle crash. The nurse assesses a client trapped under a car who is apneic and has a weak pulse at 120/min. After repositioning his upper airway the client remains apneic. Which of the following actions should the nurse take? Place a black tag on the client's upper body and attempt to help the next client in need 22.A nurse is triaging clients injured during a tornado. The nurse assesses a client who has an open fracture of his arm. Which of the following actions should the nurse take?Place a yellow tag on the client's upper body 23.A nurse is the triage officer in the emergency department when four clients arrive following a factory explosion. which of the following clients should the nurse care for first? An unconscious adult client who has a sucking chest wound, respirations of 38/min, and capillary refill of <2 seconds 24.A nurse is educating community members about how to prepare for disaster. Which of the following supplies should the nurse instruct the clients to include in a disaster preparedness kit? (Select all that apply) Clean clothing Personal identification Matches Prescription medications 25.A nurse is assisting with triaging clients following an explosion. Which of the following clients should the nurse identify as the highest priority? A conscious adult client who has second-degree burns on both lower legs; respiratory rate is >30/min 26.A nurse is reviewing the guidelines for reporting nationally notifiable infectious diseases. Which of the following diseases should the nurse plan to report to the Centers for Disease Control and Prevention (CDC)? Lyme disease 27.A community health nurse is reviewing the levels of disease prevention. Which of the following activities is an example of tertiary prevention? Providing treatment for clients who have chronic obstructive pulmonary disease 28.A charge nurse is discussing staff nurses' responsibilities in preplanning for response to a disaster. Which of the following responsibilities should the nurse include in the discussion? Identify community resources that are available 29.A nurse is reviewing treatment protocols for clients exposed to bioterrorism agents. For which of the following agents should the nurse plan to administer a vaccine following exposure? Smallpox 30.A nurse is providing staff education about smallpox as a bioterrorism threat. Which of the following statements indicates an understanding of this agent? (Select all that apply) "Smallpox is transmitted person to person" "Naturally occurring smallpox has been eradicated from the world" "Smallpox is often confused with varicella" 31.A nurse is preparing a response protocol for botulism as a bioterrorism agent. The nurse should prepare the protocol based on which of the following information? (Select all that apply) -Botulism can produce paralysis within 12-72 hr following exposure -Vomiting and diarrhea are expected findings following exposure32.A nurse is preparing to care for a client who has suspected exposure to plague as a result of bioterrorism. Which of the following actions should the nurse plan to take first? Initiate contact precautions 33.A nurse is planning a staff education session regarding biological weapons of mass destruction. Which of the following should he plan to include in the session? (Select all that apply) Smallpox Anthrax Botulism 34.A charge nurse is discussing the phases of community response to disaster with nursing staff. Which of the following statements indicates an understanding of the heroic phase of disaster response? Personnel are willing to work in dangerous conditions to provide assistance. 35.A community health nurse is providing a community education program about disaster preparedness. Which of the following should the nurse recommend that clients include in their family's disaster readiness supply kit or "go bag"? (Select all that apply) Pencil and paper Whistle Antibiotics Copies of insurance cards 36.A nurse is reinforcing teaching with a group of adolescent females who are pregnant about expected changes related to pregnancy. Which of the following client statements indicates understanding of the teaching? It is normal to have white vaginal discharge 37.A nurse is leading a therapeutic group for clients at an outpatient mental health clinic. Which of the following client statements indicates a problem with role transition? I just can't seem to find any energy to take care of my children since my husband divorced me 38.A nurse is preparing to administer ciprofloxacin to a client. The nurse should identify that the medication is treatment for exposure to which of the following agents? Anthrax 39.A nurse is caring for a client who suspects recent exposure to inhalation anthrax. Which of the following findings indicate possible exposure? Flu-like symptoms 40.The Nurse-Led Wellness Center is a type of nursing center that focuses on: A. Health-promotion, disease-prevention, and disease-management programs B. Programs aliated with major for-prot health corporations C. Programs for special populations and specic health conditions D. Public health programs 41.A newly graduated nurse is looking into applying for community-related jobs. Among all the possibilities are the following, EXCEPT: A. School NursingB. Occupational Nursing C. Trauma care Nursing D. Home care nursing 42.A riverfront community builds a retaining wall to divert floodwater away from the town. Which term describes what has happened (related to disaster)? Mitigation 43.Which levels of intervention are nurse-led centers designed to provide? Primary, Secondary, and Tertiary prevention 44.The new community nurse is interested in serving rural communication. After exploring different possibilities, which one do you think the nurse chose? Telehealth 45. A nurse is caring for the community as the client. Which of the following is most likely the focus of the nurse’s care? A. Providing care for aggregates living in the community B. The collective good of the population C. The provision of care for families in the home setting D. Providing health education in the community 46.The community health nurse is assigned to work in an X community. One of the responsibilities for the nurse is to investigate if that is a rural, urban or suburban community. (This is important for the assessment). She must have to look for all the following data, EXCEPT): A. Distance between the community and the core city. B. How many business and medical services offices are located on the community assigned. C. The demographic data, especially how many people are residents in the assigned community. D. The time needed to commute from the assigned community to the center of the city 47.A nurse is completing a community assessment. Which of the following actions would be most likely for the nurse to complete? Determine the weaknesses of a community Assess individual needs within a community Perform the core functions of public health nursing Identify community needs and clarify problems 48.A nurse is working in a community that is experiencing the Honeymoon Phase after a disaster. Which of the following is characteristic of this phase? Survivors share their stories 49.The best definition of “Doughnut Effect” is: A. when the people and people activities shift out from the center of the city to the suburbs. B. When the people move out near the Donuts stores. C. When the people of the city move out the center to work in business outside the downtown.D. When people shift from the rural environments to the core center of the cities. When the people of the city move out the center to work in business outside the downtown. 50.In Community Health Nursing and Public Health, the best description of a rural environment is: A. The rural environment is called to the location where low-income families live. B. Is there no neighbors or business oces near your home, you live in a rural environment. C. Communities having fewer than 20,000 residents or fewer than 99 persons per square mile; or it may be described in terms of the distance D. When your town is a small village with 15 residents and take 10 minutes to get the downtown area Community having fewer than 20, 000 residents or fewer that 99 persons per square mile; or it may be described in terms of the distance 51-A nurse is participating in the preparedness stage of disaster management. Which of the following describes what is happening? A. Heightened inspection and increased security in the community B. Incorporation of provision of pets into local disaster plans C. Purchase of personal protective equipment for all citizens D. Assembly of disaster kits for the home, workplace, and car 52- A nurse responds to a natural disaster. Which of the following is the nurse most likely responding to? A. Transportation accident B. Pollution C. Communicable disease epidemic D. Fire 53- A riverfront community builds a retaining wall to divert flood water away from the town. Which term describes what has happened? A. Community resilience B. Rapid needs assessment C. Mitigation D. A natural disaster 54- 10. A nurse is triaging victims of a multiple motor-vehicle crash. The nurse assesses a client trapped under a car who is apneic and has a weak pulse at 120/min. After repositioning his upper airway, the client remains apneic. Which of the following actions should the nurse take? A. Start CPR B. Place a red tag on the client’s upper body and attempt to help the next client in need C. Place a black tag on the client’s upper body and attempt to help the next client in need. D. Reposition the client upper airway a second time before assessing his respirations 55- 5. The nurse is participating in a biological disaster simulation where citizens are exposed to pneumonic plague. Which of the following interventions should the nurse plan to use while caring for these clients?A. Initiate airborne precaution B. Administer an antitoxin C. Initiate droplet precautions D. Destroy the lines after use 56- Which levels of intervention are nurse-led centers designed to provide? Local, state, and national needs Individual, family, community needs Primary, secondary, and tertiary prevention Uninsured, under-insured, and insured populations NURS 403/2020 CUARTA PARTE 1. A nurse is providing staff education about smallpox as a bioterrorism threat. Which of the following statements indicates an understanding of this agent (SATA) A. "Smallpox is transmitted person to person." B. "Infection is characterized by severe respiratory distress." C. "Smallpox vaccination ensures lifelong immunity." D. "Naturally occurring smallpox has been eradicated from the world." E. "Smallpox is often confused with varicella." 2. A nurse is caring for a client following exposure to inhalational anthrax due to bioterrorism. Which of the following medications should the nurse prepare to administer? (SATA) A. Ciprofloxacin (Cipro) B. Doxycycline (Vibramycin) C. Amoxicillin (Amoxil) D. Penicillin G (Bicillin) E. Cefotaxime (Claforan) 3. A nurse is preparing a response protocol for botulism as a bioterrorism agent. The nurse should prepare protocol based on which of the following? (SATA) A. Botulism can produce paralysis B. The Centers for Disease Control and Prevention (CDC) should be notified when more than three cases are suspected C. Botulism is acquired through contact D. Vomiting and diarrhea are expected findings E. Botulism is a toxin found in castor beans 4. A public health nurse has just completed an educational program community who may have been exposed to agents of bioterrorism one of the nurses who attended the program regarding the clinical indicates a need for further education?A. "Smallpox has been considered eradicated worldwide." B. "People who were immunized against smallpox before to be protected." C. "It is easy for providers to distinguish smallpox from present early with the disease." D. "Unlike chickenpox, the vesicles of smallpox are m 5. A nurse is preparing to care for a client who has suspected exposure to plague as a result of bioterrorism. Which of the following actions should the nurse plan to take first? A. Prepare for drainage of lesions B. Initiate contact precautions C. Notify the CDC D. Decontaminate the client 6. A nurse in an emergency department is caring for a victim of a suspected...exposure to cutaneous anthrax. Which of the following is an expected...? A. Immediate onset of respiratory distress B. Flu-like symptoms 48 hr after exposure C. Itching of the skin progressing to ulceration over 1 to 7 D. Immediate onset of vascular lesions of the skin 7. A nurse is reviewing treatment protocols for clients exposed to bioterrorism agents. For which of the following agents should the nurse plan to administer a vaccine following exposure? A. Anthrax B. Botulism C. Plague D. Smallpox 8. A nurse is assessing a client who is experiencing difficulty sleeping, having nightmares, experiencing feelings of survivor guilt, and having difficulty concentrating one year after the death of a relative during a destructive and devastating natural disaster. Which of the following describe the client's symptoms? A. Generalized anxiety disorder B. Post-traumatic stress syndrome C. Historionic personality disorder D. Dissociative identity syndrome 9. A nurse is working with community members in order to aid them in preparation for any disaster. She helps the clients compile a personal list of basic supplies needed in case of a disaster. Which of the following should the nurse instruct the clients to include? (SATA) A. Three quarts of water per person B. Clean clothing C. Personal identification D. Matches E. Prescription medications 10. A nurse is working with an interdisciplinary committee in the development of a community-wide emergency response plan in the even of a disaster. The nurse should assure that the community's emergency communication plan for a non-biological or chemical incident includes which of the following agencies to be notified immediately following the call to 911? A. Office of Emergency Management OED B. The Federal Emergency Management AgencyC. American Red Cross D. U.S. Department of Homeland Security 11. A nurse is triaging clients after a local disaster. Which of the following clients should the nurse recommend to discharge first? A. A client who has a large hematoma over the left eyebrow and is reporting a headache B. A client who has a contusion to the upper right abdominal quadrant and is reporting pain C. A client who has a suspected fracture to the right wrist and is reporting limited movement D. A client who has ecchymosis to the left ankle and is reporting swelling 12.A triage nurse is implementing the three-tier triage system to test an organization's disaster preparedness. Which of the following is an example of the emergent tier level? A. The client who has a punctured femoral artery B. The client who displays multiple fractures C. D. 13. A hospital is participating in a disaster simulation in which a toxic substance is released into a crowded stadium. Which of the following is the lowest priority for a nurse? A. B. C. Educating the client and family D. Maintaining calm amidst the chaos 14. A nurse is discussing community responses to disaster with nursing staff. Which of the following statements indicates an understanding of the heroic phase? A. "Personnel are willing to work in dangerous conditions to provide assistance." B. "Survivors come together and share stories of survival." C. "Responders experience exhaustion due to extended relief efforts." D. "Normally begins to return to the community." 15. A nurse is participating in a biological disaster situation where citizens are exposed to pneumonic plague. Which of the following isolation precautions should the nurse plan to use while caring for these clients? A. Airborne B. Contact C. Droplet D. Protective 16. A nurse is discussing emergency response with a newly licensed nurse. The nurse should identify which of the following is a triage officer during the time of a disaster? A. Members of the Federal Emergency Management Agency (FEMA) B. Responding law enforcement officers C Representatives from the American Red Cross D. Nurses and other emergency medical personnel 17. A nurse is assisting with disaster triage following a bomb explosion in a bus station. Which of the following clients should the nurse identify as being the highest priority?A. An unconscious adult who has a large head wound with gray matter exposed; absent respirations; capillary refill > 2 seconds B. A conscious adult with second degree burns on both lower legs; respiratory rate is > 30/min; capillary refill is < 2 seconds C. An unconscious 6-month-old infant with no respirations; no visible injuries; no pulse D. A conscious adult wearing a medic-alert diabetic bracelet; respirations are < 30/min; capillary refill is < 2 seconds, and client reports feeling clammy and shaky 18. A community health nurse is providing a community education program about disaster preparedness. Which of the following should the nurse recommend to include in an individual personal readiness supply kit or "go bag"? (SATA) A. Pocket knife B. Whistle C. Antibiotics D. Bank account information E. Household bleach 19. A nurse is working at a health care facility in which there is a threat of flooding, and an internal disaster plan has been activated. Which of the following clients should the nurse evacuate first? A. A client receiving platelets for late-stage leukemia B. A client who has a saline lock device admitted with chest pain C. A client receiving antibiotic IV therapy and wound vacuum care D. A client who has a casted arm and balanced suspension traction on a lower extremity 20. A nurse is discussing with nursing staff the nurses' responsibility in preplanning for a response to a bioterrorism disaster. Which of the following should be included in the discussion? A. Identify community medical and social services that will be available B. Evaluate the impact of a disaster on the community C. Assess survivors of a disaster for levels of psychological stress D. Link victims with support agencies to help with food, clothing, shelter, and counseling needs 21. A charge nurse is discussing disaster response with the nursing staff. Which of the following statements indicates staff understanding of the Hospital Incident Command System (HCIS)? A. "HCIS ensures that necessary antibiotics and antidotes are available." B. "HCIS is focused on having multi-disciplinary responders available." C. "HCIS identifies facility responsibilities and channels for reporting." D. "HCIS provides additional responders when needs exceed the ability of local or state agencies." 22. A nurse is discussing the requirements of the Joint Commission for accreditation regarding disaster planning. Which of the following statements indicates a need for further teaching? A. "A disaster drill needs to be held at least once a year." B. "An actual disaster can take the place of a disaster drill." C. "Simulated patients need to be used for a disaster drill." D. "The hospital should use community resources for a disaster drill." 23. A bomb exploded at a local business office building. Patients are starting to arrive at the hospital. Which patient would you categorize in the red (emergent) category? A. A patient that comes walking in to the ER with a head contusion B. A patient that is not breathing and has multiple traumatic amputationsC. A patient who has rapid respirations, a delayed capillary refill, can follow a simple command and with interventions could survive D. A patient that is having an anxiety attack 24. The START triage tool was developed to be able to triage adult patients within 30 seconds and is the most commonly used tool in the United States. What are the 5 basic parameters to this tool? (select all that apply) A. The ability to walk B. The presence or absence of spontaneous respirations C. The respiratory rate D. An assessment of perfusion E. The ability to obey commands 25. An internal disaster is classified as an event that occurs within an organization or facility (it poses a threat to disrupt the environment of care). Which of the following are examples of internal disasters? (Select all that apply) A. Fire B. Tornado C. Hurricane D. Loss of utilities 26. Disasters have been further classified according to the magnitude of the disaster in relation to the ability of the organization or community to respond. How many levels are there? A. Two B. Six C. Ten D. Three 27. In the field of a disaster, there are different triage zones that are classified. Which of the following is considered a hot zone? A. A distance of 300ft from outer perimeter of the hot zone. Rapid triage takes place. Only minimal amount of treatment is rendered to provide essential stabilization. B. An area immediately adjacent to the incident, minimal triage and medical care in this area and all staff must be in protective gear C. Adjacent to warm zone, decontaminated patients enter, secondary triage is performed. 28. The chain of communication is an important part of disaster management whether it be a terrorist threat or a natural disaster. What areas will always be notified in the case of a terrorist threat? A. Office of the president, FEMA, HHS, and CDC B. State office of emergency, office of the president, and the state governor's office C. CDC, State Department of Health, and the State Office of Emergency D. None of the above 29. What is most crucial element in disaster management? A. Tell everyone that everything will be okay B. Plan ahead (do not wait until disaster happens)C. Wait until a disaster happens and then plan according to the disaster D. Utilize the same plan for every possible disaster 30. You are working in the hospital as a nurse manager. An earthquake just occurred and your hospital will be receiving multiple patients. It is important to do which of the following? (Select all that apply) A. Call in additional staff B. Designate staff roles the day of the disaster C. Prepare for influx of patients D. Prepare STAT charts once patients start arriving 31. How many tiers of triage are there within a hospital on a day to day basis? A. Four (mentation, pulse, respirations, ambulatory status) B. Three (emergent, urgent, non-urgent) C. Two (red, black) D. One (treat everyone) 32. A patient comes into the ER after a disaster. He is not responsive and is not breathing and no pulses can be found. You are the triage nurse, what color tag should this patient receive? A. Red B. Blue C. Green D. Black 33. The nurse is aware that all of the following can contribute to a mentally ill patient in the aftermath of a disaster except? A. Exacerbations of pre-existing chronic mental disorders B. Severe stress C. Most vulnerable people D. PTSD 34. The student is discussing the differences associated with grief, mourning, bereavement, and depression to a patient. Which of the following statements should the charge nurse intervene in when the student states what to the patient? A. Grief is an internal, emotional response to loss, affliction, or regret. B. Bereavement is an external expression of grief as seen in traditional or creative rituals, especially for the dead. C. Depression is a state of feeling sad or, more specifically, is an emotion and concentrating, and feelings of dejection. D. The charge nurse doesn't have to intervene because the student conducted proper teaching to the patient. 35. An environmental disaster, such as a oil spill, can result in what? (SATA) A. Rioting and Looting B. A increase in demand for oil C. Sense of mistrust toward the government and businesses D. Leads to sense of hopelessness and helplessness in citizens affected C, D36. A few weeks after a tornado, a nurse is assessing a survivor and he tells the nurse he has accepted that a tornado did occur and has recently joined a new CrossFit group, and started volunteering at the Red Cross. According to his responses the nurse can place him in what phase of emotional recovery outlined by the American Red Cross? A. Honeymoon Phase B. Disillusionment phase C. Reconstruction Phase D. Heroic Phase 37. What is one of the leading causes of morbidity and mortality in displaced pediatric population? A. Upper respiratory Infection B. Car accidents C. Ingestion of various medication D. Uncontrolled Diabetes 38. A worried mother, is talking to a nurse about her son's condition after being exposed to nuclear radiation. The mother begs the nurse to see her son. Select the appropriate response by the Nurse. A. Sure, it is your right as a mother. B. I Cant imagine what you are going through. C. You can see your son when he is stabilized and decontaminated. D. You can see your son after he is bathed, and in new clothes. 39. Which of the following is considered a Natural Disaster? A. Bombing B. Earthquake C. Influenza D. Harmful Chemicals 40. Wildfires can quickly spread out of control, causing extensive damage to personal property and human life. Which of the following are classified as wildfires? (Select all that apply) A. Crown Fires B. Air Fires C. Surface Fires D. Ground Fires 41. Air pollutants can cause health damages, what are the types of health damages? Select all that apply. A. Immune B. Neurological C. Cardiovascular D. Respiratory 42. What organization is responsible for research and setting national standards for a variety of environmental programs? A. The CIA B. Environmental Protection AgencyC. Agency for Healthcare Research and Quality D. Health Resources and Services Administration 43. Which of the following Category A agents require BSL-4 laboratory facilities? A. Smallpox and plague B. Anthrax and tularemia C. VHF and smallpox D. Botulism and plague E. Smallpox and anthrax 44. Which of the following is the natural reservoir for Ebola virus? A. Small rodent B. Mosquito C. Green monkey D.Tick E. The natural reservoir is unknown 45. Nerve agents are well absorbed through the skin. A. True B. False 46. There is a Donald Trump protest starting to form at a nearby mall. Protesters are starting to become violent and rowdy. Police officers are called onto the scene and they are preparing to intervene. Which chemical agent are they prepared to use against the rowdy crowd? A. Blister Agents B. Blood Agents C. Nerve Agents D. Riot Control Agents 47. What is the most effective way to break the chain of infection? A. Immunizations B. Frequent Doctors visit C. Showers D. Good Hand Hygiene 48. Which of the following are the leading causes of infectious diseases deaths? Select all that apply. A. Tuberculosis B. HIV/AIDS C. Malaria D. Herpes NURS 403/2020 QUINTA PARTE. 1.Anthrax is a form of bacteria and is responsive to treatment with ciprofloxacin, (Antibiotic fluoroquinolones) which is the proffered antibiotic of treatment along with doxycycline for both inhalation and cutaneous AnthraxSx: Flu like Sx, HA, Fever and chills, weakness Treatment 60 days if bioterrorism 10 days if inhalation 7-10 days cutaneous 2.The nurse should administer an antibiotic and the Anthrax vaccine within 24 hrs as prophylaxis to all clients exposed to Anthrax and are not exhibiting manifestations of illness. 3. Botulism: Produce paralysis within 12 to 72 hours following exposure Sx: Vomiting and diarrhea Treatment: airway with possible mechanical ventilation Adm of ANTITOXIN Elimination of Toxin: inducing vomiting, enemas, surgical excision of wound tissue 3. Smallpox: The rash in smallpox is more centrifugal than chickenpox in its presence of lesions. The lesions appear monthly on the face and extremities. 4.Chickenpox lesions are centripetal with lesions more frequently found on the trunk of the body. 5. Plague: Pneumonic plague: Droplet precaution Sx: Fever, HA, weakness, pneumonia w/ SOB, CP, cough, bloody or watery sputum. Dropper precaution till 72 hrs. after antibiotics 6.Plague: Contact precaution should be initiated prior to care for a patient suspected exposure to a plague as a result of bites from infected fleas due to a risk of transmission. 7. Tularemia toxicity: No vaccine able. TTo Streptomycn & gentamicin. Aminoglycoside Ototoxic and nephrotoxic. Tinnitus 8.Situtional role change: Problem in role transition can result from a change in personal, occupational or social status. Ex: A person lack of energy while dealing with a problem. Temporary: resume the role when the illness end Permanent: previous role are no longer able. 9.Disaster preparedness occurs at the national, state, and local levels. Personal and family preparedness are crucial components of disaster preparedness, as is professional preparedness for individuals employed in civil service and health care. 10. Family disaster readiness supply kit or go bag: Pencil and paper, whistle, copies of ins cards, household bleach 8.During the heroic phase responders are willing to work in dangerous conditions for long periods of time to provide assistance. 11.A nurse who works with an interdisciplinary disaster committee to develop a community-wide emergency responsive plan in the event of a non-biological or chemical incident should include the agency OEM ( Office of Emergency Management) in the community communication plan to becontacted following the call to the emergency communication center (911). This agency is responsible for determining if additional resources to the initial first responders are needed. 12. How to prepare for a disaster: Basic supplies. 3.7 littlers (1 gallon) of water per person per day, a three day supply is recommended, clean clothing, personal identification, matches, prescription med 13.Primary prevention addresses the needs of healthy clients to promote health and prevent disease with specific protections. It decreases the risk of exposure individual/ community to disease. Immunization programs Child car seat education Nutrition, fitness activities Health education in schools 14.Secondary prevention focuses on identifying illness, providing treatment, and conducting activities that help prevent a worsening health status Communicable disease screening, case finding Early detection, treatment of diabetes mellitus Exercise programs for older adults who are frail 15.Tertiary prevention aims to prevent the long-term consequences of a chronic illness or disability and to support optimal functioning. Begins after an injury or illness Prevention of pressure ulcers after spinal cord injury Promoting independence after traumatic brain injury Referrals to support groups Rehabilitation center 16. Nursing intervention: PROMOTE HEALTHY LIFESTYLE BEHAVIORS 17.CLIENT WILL WALK 30 MINUTES 5 DAYS A WEEK. CDC RECOMMENDATIONS INCLUDE ENGAGING IN A MODERATE EXERCISES, SUSH AS WALKING, FOR A TOTAL OF 150 MINUTES EACH WEEK 18. Older Adults: health concerns leading to cause death: Chronic Obstructive pulmonary disease: Emphysema and Chronic bronchial NO ASTHMA 19.National health Goal: THE GREATER RISK FOR AN OLDER ADULT WHO IS ABUSE BY THE FAMILY IS FURTHER INJURY FOR CONTINUE ABUSE, THEREFORE, THE FIRST ACTION THE NURSE SHOULD TAKE IS TO REPORT THE FINDINGS TO THE SUPERVISOR. NURSE ARE REQUIRED TO REPORT SUSPECTED CASES OF CHILD AND OLDER ABUSE. 20. Breast examination palpate the incisional lines(mastectomy). Techniques: Inspection and palpitationself inspect: in front of a mirror and palpate them during a shower with soapy hand. Optimal time is right after menstruation ends. 7-10days. Client pregnant or postmenopausal should do it breast self-examination (BSE) on the same day each month. No BP, no IV, same for fistula for dialysis NO in that side. 21.CLIENTS WHO HAS ASTHMA AND COPD, emphysema ARE SPECIALLY VULNERABLE TO OZONE EFFECTS. Avoid dog hair, etc The ozone exerts its primary adverse effects on the respiratory system, reducing lung function and increasing the risk of respiratory infection. 22-health risk- modifiable risk- Tabacco 23.Older expected physiologic changes Increased susceptibility to glare, greater difficult in seeing at low levels of illumination, and alterations in color perceptions Decreased cough reflex, increased airway resistance, fewer alveoli, and greater risk for respiratory infections Decreased bladder capacity and a reduction in renal blood flow Increased systolic blood pressure, thickening of blood-vessel walls, and decreased peripheral circulation Dehydration of intervertebral discs decreased muscle strength and mass, and decalcification of bones. 24.When an older adult live alone, he/she is unable to drive him/herself to the store and is afraid to cook on the stove the nurse should recommend Meals and Wheels services that is a service that delivers meals daily to older adults who need them either at seniors centers or directly to their homes. ¨Esta pregunta está en el quiz segunda parte y no la puso en el midterm¨. 25. Injury prevention: ensure adequate lighting (pt durmiendo en la noche poner luz para caminar en la noche) and wear eyeglasses and hearing aids 24.Determine 4 levels: 1) Emergent/Class I/Red 2) Urgent/Class II/ Yellow 3) Non-urgent/ Class III/ Green 4) Expectant/ Class IV/Black 26.Emergent, Class I: Red Immediate threat to life. Do not delay care. Urgent, Class II: Yellow, Major injuries that require treatment. Can delay treatment for 30 min - 2 hours. Non-Urgent, Class III: Green, Minor injuries that do not require immediate treatment. Can delay treatment for 2 -4 hours. Expectant, Class IV: Black, Expected and allowed to die. Transfer to morgue.27.Any adult who has RR over 30/min as well as if the patient is unconscious, which constitutes AMS requires immediate attention 28. Malpractice: wrong medication, a nurse administer large doses due to calculation error. Client has cardiac arrest and die. 29. compression tube due hemorrhage SENGSTAKEN BLAKEMORE internal ballon 30. Check placement: Aspirate gently to collect gastric contents. Testing pH (4 or less is expected), and assess odor, color, and consistency 31. after placement verification. Confirmation placement with X ray if the tube is not in the stomach, advance it 5cm (2in) and repeat the placement check 32.THE POSTERIOR TIBIAL IS LOCATED ON THE INNER ANKLE, ONE-THIRD OF THE WAY ALONG A LINE BETWEEN THE TIP OF THE MEDIAL MALLEOLUS (END OF THE TIBIA) AND THE POINT OF THE HEEL. IT IS MOST EASILY PALPATED ABOUT 2.5 CM HIGHER, WHERE IT RUNS BEHIND THE MEDIAL MALLEOLUS. 33.Diaphragmatic breathing for a client who is going to have a surgery: Teach the client to Inhale slowly and evenly through the nose until chest expansion is maximized. DO NOT LET THE PT KNOW YOU ARE CHECKING RESPIRATION if you are checking RR 34.IMPORTAN T NOTE Aortic: Just right of the sternum at the second ICS Pulmonic: Just left of the sternum at the second ICS Erb’s point: Just left of the sternum at the third ICS Tricuspid: Just left of the sternum at the fourth ICS Apical/mitral: Left midclavicular line at the fifth ICS, below the left nipple line 7.6 cm (about 3 in) to the left of the sternum. 35. When a nurse is taking care of a patient who need stool specimen collected the nurse should label the specimen container and place it in a clean, plastic biohazard bag which is also labeled. This ensures proper identification and prevents contamination and spillage during transport. 36.When a nurse is taking care of a client that who is increasingly restless and intermittently confused, she should move the client to a room closer to the nurses' station. This will make it easier for the staff to observe the client, should the client behave in an unsafe manner. 37.When a nurse is caring for a client who has peripheral vascular disease and reports difficulty sleeping because of cold feet the nurse should obtain a pair of slipper-socks for the client. In cold weather or when the client's feet are cold, he/ she should wear extra socks or slipper socks to help provide warmth and increase his/her level of comfort. 38. Contact precaution required a private room or a room with other client who have the same infection, gloves and gowns worn by the caregivers and visitors, disposal of infection dressing materials into a single, nonporous bag without touching the outside of the bag 39. AIRBONE Precaution: <5mcg, mask N95 or HEPA if TB, negative pressure, if splashing wear full face. Laryngeal TB and Pulmonary TB Measle, Varicella: Private room, Mask, respiratory protection device for CAREGIVER and visitors.  Everyone entering or leaving the room should close the door behind them.  Nurse must always disposal of needles and sharp instruments in puncture proof sharps containers 40-Very important: ICF and ECF = Homeostasis Fluid can move between compartments diffusion, active, transport, filtration, osmosis. 41. Fluid Imbalance: ICF and ECF (intravascular-plasma, interstitial-around cel, pericardial, lymph, pleural, pancreatic, intraocular, biliary, peritoneal, synovial Fluid Volume Deficit: 42.Isotonic FVD loss water and electrolytes from ECF= hypovolemia because intravascular fluid is loss 43. Dehydration: loss water without electrolytes then increase Hct, serum e- and urine specific gravity 44. FVD can lead shock hypovolemic 45.Older adults have increase the risk decrease in total body mass, total body water, decrease in the ability to detect THISTY. Hypovolemia=Isotonic 46. Nursing care: For fluid replacement Adm IV fluid a prescribed Isotonic solution such as lactated Ringer or 0.9% sodium chloride, blood transfusion. 47. Fluid Volume excess: interstitial to plasma fluid shifts (hypertonic fluids, burns) 48. nursing care: Monitor I&O and check capillary refill (less than 2 sec) and level of conscious 49. Hyponatremia fluid moves ECF into ICF cell swollen cerebral edema: water excess in the plasma or lost Na rich fluid. 50.Expects finding in Hyponatremia: Neuromuscular, physical assessment (normal, dec, or increase ECF volume) and VS. 51. Acute hyponatremia: Adm hypertonic oral and IV fluid 3% sodium chloride. 52. Hypernatremia: A client who has an elevated sodium level and is NPO requires a hypotonic IV solution, such as 0.45% sodium chloride or 0.225% sodium chloride. 53. Na ECF= brain K ICF= Heart Na: 136-145 K 3.5 to 5.0 54. hypokalemia: ECG and GI 55.IV potassium supplementation mixed by a pharmacist and double check by two nurses prior to Adm 56.Nursing Process: The assessment phase includes asking the client about her health history, physical concerns and health care expectations. A review of possible allergies is included in the assessment phase. 57.Neurological assessment focus priority if it is an appropriate nursing intervention when a client displays sudden confusion. Sensory alterations can occur when a client is experiencing multiplesensory stimuli and can result in inappropriate sensory responses. Tolerance to stimuli may be affected by fatigue and emotional and physical well-being. 58. If pt has respiratory problem then focus in Respiratory assessment. FOCUS Priority on the chief of the patient. 59.A client who has a decreased level of consciousness and an inability to swallow is at risk for aspiration. Lying flat also increases this risk. The priority action by the nurse is to keep the head of the bed elevated 30o to 45o to promote gastric emptying and reduce the risk of aspiration. 60.Isometric exercises involves static (no movement) contraction of a muscle without any movement of the joint. Isometrics promote increased muscle mass, strength, and tone for clients who are on bedrest. Nurse has to instruct the client to tighten muscle groups for a short period, and then relax. 61.The priority assessment the nurse should perform when a client arrives to ER with any cut in his/her body is to determine whether the client will require a tetanus immunization by identifying the date the client last received one. Client is at risk for Clostridium tetani; An adult should have a tetanus booster immunization every 10 years and after any severe or dirty wound. TD  DTaP born to 7years old (0nce)  Tdap booster adolescent 11 years old, 19-64, pregnant 27-36 weeks of gestation (0nce)  Td every 10 years. 62.The nurse should use a gait belt to help support the client during ambulation. A gait belt helps keep the client’s center of gravity stable and helps maintain balance and prevent falls. 63. Crash motor vehicle open fracture femur: yellow/urgent/class II 64. 1st RR>30 mental status alterado: Primer priority. Primer client to give service. 65.Examen to four clients: swollen gum, pruritus, urinary problem, dysphagia. Which is priority? Dysphagia 66. Pediculosis: Nit hatch up to 7-10days 67 Biologic diseases: Smallpox, anthrax, botulism. 68 Client que viene con pies fríos? Mirar 37 Midterm AM 2020 Midterm de community 1. Which of the following best describe the Public Health? The disordered physiological processes associated with disease or injury. The study of the development, structure, and functioning of human society. Includes the study of incidence, distribution, possible control of the diseases, statistics, assessment of the cultural and behavioral factors of the society as a collective. The scientific study of the human mind and its functions, especially those affecting behavior in a given context.2. Community Health Nursing is a community-based practice. Which best explains this statement? The service is provided in the natural environment of people The nurse has to conduct community diagnosis research to determine nursing needs and problems The service is based on the available resources within the community Priority setting is based on the magnitude of the health problems identified 3. One of the major reasons for the increasing of care provided in a community-based setting include: The continuous increasing of the population in the countryside Increasing community-based nursing jobs. The high cost of community urgent care. The growing cost of hospital care. 4. Memory, understanding, and reasoning as part of the psychomotor domain in education. True False 5. A community health nurse visits a homeless shelter to provide directly observed therapy (DOT) to several clients who have been diagnosed with TB. Which of the following best describes the rationale for this nursing intervention? Homeless clients do not care about their health Poor adherence can result in drug resistance Homeless clients do not have access to medications These medications are so powerful, clients must be observed for reactions 6. Which of the following describes the consequence of the successful implementation of the Affordable Care Act? Americans will pay closer attention to their health status Most of the population will be covered by insurance Public health departments will need to increase the number of nursing positions The prevalence of obesity will decrease 7. The development of the group includes the adjourning phase. True False 8. A client is receiving PHC services. Which of the following expectations would the client have?The care would be provided by a physician Private insurance would be necessary to receive care The emphasis of care provided would be on prevention A referral would be made to a specialist after receiving services 9. A nurse who is working in a community setting applies evidence-based practice when planning community health education. Which of the following should the nurse use in order to enrich the application of this evidence? Qualitative studies Randomized clinical trials Multiple research methods Quantitative studies 10. Which statement about feminist ethics is correct? Feminists include only women in their worldview. Women’s thinking and moral experiences are important and should be considered. Persons who ascribe to feminist ethics are passive and wish to pursue their ideas through the legislative process. Feminists believe that men should not be nurses. 11. One of the defining characteristics of Public Health Nursing is its emphasis on practice. It’s based on: strategies for treat patients in rehabilitation stages. strategies for health promotion and disease prevention strategies for prescribe screening to the maintenance of health. none of the above 12. The National Prevention Strategy targets interventions in multiple settings to move the health of the population from sickness to prevention of health and illness promotion. True False 13. Which of the following is an example of a public health agency? Federal Bureau of Investigation (FBI) Drug Enforcement Administration. (DEA) Women, Infants, and Children (WIC) Medical Center Clinics (MCC) 14. A PHN is implementing interventions at the systems level of practice. Which of the following interventions would be used by the nurse?Identify health problems in the community Involve the entire community in solving the health problem Provide outreach services to populations at risk Change laws, policies, and practices that influence population-based issues 15. PHNs utilize registries to identify children with delayed or missing immunizations. They subsequently follow up with families by phone calls or home visits. Which of the following levels of practice is being implemented? Individual Community Systems Policy 16. A nurse uses Health People 2020 as a guide when planning health education in the community. Which of the following actions would be taken by the nurse? focus on avoiding cigarette smoking and using alcohol in moderation educates clients using primary and secondary levels of prevention use Bloom’s taxonomy when planning educational objectives design health fairs aimed at individuals 17. Community Nurses who provided care to people in their homes were called _____ nurses. visiting private duty hospitalist duty critical home 18. In an educational process to the community, the nurse should know that the easiest thing to change is the knowledge. True False 19. A nurse plans to contact a multilateral organization. Which of the following would the nurse most likely contact? United Nations (UN) Project Hope Catholic Relief Services Baptist Charity Group20. A nurse is involved in identifying individuals with unrecognized health risk factors or asymptomatic disease. Which of the following public health interventions is being applied? Referral and follow-up Health teaching Surveillance Screening 21. The practice focus of a Public Health Nursing practice is: The family as a group The person as a unitary individual The group as a part of the society The community as a whole 22. Which step, when working with the community organization to make changes, involves training of potential leaders in the community? Core group formation Integration Community organization Community study Core group formation 23. A social marketing campaign urging community members to avoid driving motorized vehicles after consuming alcohol is implemented in a local community. Which of the following levels of practice is being demonstrated? Individual Systems Community Government 24. The factors, exposures, characteristics, and behaviors that determine patterns of disease are described using: descriptive epidemiology. analytic epidemiology. distribution determinants 25. A nurse is examining the income levels and social networks of individuals in a community. Which of the following is the nurse investigating? CultureDeterminants Development Hobbies 26. A PHN uses Assumption 2, “Public health nursing practice focuses on populations,” to guide practice. Which of the following would be considered a population of interest? Healthy school children Homeless individuals A person recently diagnosed with diabetes Teenage parents 27. Florence Nightingale’s contributions to public health included: caring for the sick, poor, and neglected in institutions and at home using a population-based approach that led to improved environmental conditions writing the Elizabethan Poor Law to guarantee medical care for all founding of the district nursing association to provide health care to needy people 28. A nurse is teaching a client diagnosed with gonorrhea how to prevent reinfection and further spread. Which of the following describes the action taken by the nurse? Primary prevention Secondary prevention Tertiary prevention Secondary promotion 29. The interaction between an agent, a host, and the environment is called: natural history of the disease. risk web of causality. the epidemiologic triangle. 30. Which is the primary goal of community health nursing? To support and supplement the efforts of the medical profession in the promotion of health and prevention of illness. To enhance the capacity of individuals, families, and communities to cope with their health needs To increase the productivity of the people by providing them with services that will increase their level of health To contribute to national development through promotion of family welfare, focusing particularly on mothers and children31. Which ethical principle requires “doing no harm?” Maleficence Beneficence Distributive justice Non-maleficence 32. When a Community Nurse is performing a teaching section to a group; the cognitive domain includes: the performance of skills changes in attitudes and the development of values memory, recognition, understanding, reasoning, and problem-solving memorization of one set of skills before moving on to the next 33. A nurse is teaching a group of clients newly diagnosed with diabetes how to give themselves injections. Which of the following formats would be most appropriate for the nurse to use? Health fair Lecture Non-native language session Demonstration 34. The Public Health core functions are directly linked to the healthcare promotion and prevention levels. The tertiary levels is characterized by: meets the basic needs of healthy people. the services are designed to limit the progression of the disease or disability. the function is to detect and treat diseases in an early stage. the services are designed to treat diseases in the acute stage. 35. A nurse discusses services with a federal congressman. Which of the following services would the nurse most likely be discussing? Family planning Counseling Policy making Prevention of diseases 36. A nurse offers screening for hearing defects at a local community center. Which of the following best describes the action of the nurse? Health promotion Secondary promotionSecondary prevention Primary prevention 37. One of the primaries focuses on improving the health of the American people in the twenty-first century is to address: bioterrorism and global health threats delivery of individual care and hygiene the need for increased hospital and acute care chronic disease and disability management 38. In community health nursing the model that focuses on the individual’s attitudes and beliefs toward the health is the Social Cognitive Theory. True False 39. An occupational health nurse works with an employer to develop a workplace wellness program for its employees. Which of the following levels of practice is being implemented? Individual Systems Community Government 40. The levels of practice encompassed by the Intervention Wheel are: primary, secondary, and tertiary communities, individuals and families, and systems assessment, diagnosis, and evaluation communities, populations, and aggregates 41. To understand the causes of health and disease, epidemiology studies: individuals families groups populations 42. The public health nurse is the supervisor of rural health midwives. Which of the following is a supervisory function of the public health nurse? Providing technical guidance to the midwife Providing nursing care to cases referred by the midwife 43. Which statement is true about the origins of the Intervention Wheel?It was a result of a qualitative analysis carried out by the State Boards of Nursing A panel of nurses from Iowa, Minnesota, North Dakota, South Dakota, and Wisconsin developed and refined the Intervention Wheel. It resulted from a grounded theory process carried out by public health consultants at the Minnesota Department of Health It was conceived by a group of international nurses from Norway, Kazakhstan, and Japan. 44. In 1850, the American Health Department was impacted and transformed with new policies, innovations, and collection of vital statistics. That transformation was included in _____________. Report Lindsey Report Wald Report Public Health Report Shattuck Report 45. Current threats to health in the United States that community health nurses are faced include: diphtheria, cholera, and hepatitis HIV, H1N1 influenza, and bioterrorism avian flu, tuberculosis (TB), and radiation polluted water and air 46. A nurse who was working in the 1960s used the code of ethics to guide making an ethical decision. Which code of ethics would have been used? Code for Professional Nurses Nightingale Pledge International Council of Nurses (ICN)Code of Ethics for Nurses Code of Ethics for Health Professionals with Interpretive Statements 47.The major goal of health care reform is to make a community-oriented government. Answers: True False 48.Pelvic inflammatory disease (PID) is a common complication of: Chlamydia Herpes Syphilis Gonorrhea 49. Public health professionals refer to three levels of prevention as tied to specific stages in the:surveillance process web of causation natural history of disease epidemiologic triangle 50.A person diagnosed with syphilis presents with signs and symptoms of rash, sore throat, and muscle and joint pain. Which of the following stages of syphilis is the client most likely experiencing? Secondary Primary Congenital Tertiary 51. Teaching specific patient groups requires a lot more than delivering data; the educator have to understand where your patient population is coming from. True False 52.Which of the following IS NOT a benefit of the Publich Health? The dramatic increase in number of immigrants from hispanic communities. Increase in life expenctancy Decreased number of deaths from stroke, coronary heart diseases and cancer. The use of health care budget for prevention programs. 53.Which is true of primary facilities? They are usually government-run. Their services are provided on an outpatient basis. They are training facilities for health professionals. A community hospital is an example of this level of health facilities. 54.The easiest thing to change when a public health or community health nurse develop an educational program to a person; a group or community is ________. behavior. knowledge. attitud. temperament. 55.In the United States, the majority of health care dollars are spent on: prescription drugs physician services hospital care public health services56.The educational issues; barrier to learning; or environmental conditions, are not determinants to consider when develop an educational community process. True False 57. The public health nurse (PHN) must participate in the essential services of public health. Which of the following most accurately describes one of the essential services of public health? Monitoring health status by completing a community assessment Diagnosing and investigating health problems in the world Informing, educating, and empowering people about health issues Working in law enforcement to regulate health and ensure safety 58. A nurse is exploring a job opportunity at a community health center. Which of the following expectations would the nurse have about the center? Offers comprehensive primary care services and supportive services Would be located in a large metropolitan area Provides care only to the economically disadvantaged Is used for preventive services such as flu shots 59.Which statement regarding poverty and health insurance is true? The poor in the United States are as healthy as persons with higher incomes. Millions of people in the United States are without health insurance. Persons with money or health insurance are less likely to seek health care. The poor are more likely to receive health care through private agencies. 60. Public health administrators in a community provide a health department to serve an indigent population of immigrants providing translators on certain days of the week. Which of the following best describes what is being done? Quality Libertarian philosophy Assurance Policy development 61.Which of the following best illustrates this statement? The community health nurse continuously develops himself personally and professionally.Health education and community organizing are necessary in providing community health services. Community health nursing is intended primarily for health promotion and acute treatment of diseases. The goal of community health nursing is to provide nursing services to people in their own places of residence 62.Attitudes can be change with an adequate education program and pleasant teaching-learning experience. True False Otros quizzes 1. A nurse is using basic educational principles when conducting an effective educational program. Which of the following IS NOT a principle that the nurse should use? Use a clear, succinct style Use an active voice Refer to Internet sources Use aids to highlight key points 2. A nurse is teaching a client about how to complete a wound dressing change. Which of the following conditions must be met before learning will occur? Must be able to memorize the instructions, relay this information to a partner, and demonstrate the dressing change Must master the dressing change at the time it is taught, repeat the demonstration for the nurse, and teach another person Must be able to speak the language of the nurse, have time to practice the dressing change, and master the dressing change in a short time Must have the necessary ability, a sensory image of how to carry out the dressing change, and an opportunity to practice the dressing change 3. A health educator is trying to change a client’s attitudes about smoking. Which of the following domains would be used? Cognitive Affective Psychomotor Developmental 4. A nurse uses Healthy People 2020 as a guide when planning health education in the community. Which of the following actions would be taken by the nurse? Focus on avoiding cigarette smoking and using alcohol in moderationEducate clients using primary and secondary levels of prevention Use Bloom’s taxonomy when planning educational objectives Design health fairs aimed at individuals 5. A nurse is using the educational process of selecting appropriate educational methods when planning a community health program. Which of the following steps of the nursing process does this action most resemble? Assessment Evaluation Implementation Planning 6. A nurse is teaching a group of clients newly diagnosed with diabetes how to give themselves injections.Which of the following formats would be most appropriate for the nurse to use? Lecture Demonstration Health fair Non-native language session 7. A nurse has evaluated the learning needs of a community support group. Which of following steps should the nurse take when developing an educational program for them? Consider any potential barriers to be transported. Establish goals and objectives for the program Select appropriate materials for the program Assess the dynamics of the group 8. The cognitive domain includes: changes in attitudes and the development of values the performance of skills memory, recognition, understanding, reasoning, and problem solving memorization of one set of skills before moving on to the next 9. Evidence-based public health (EBPH) is: an integration of the best available nursing expertise derived from community principles to fit the area where it is practiced a health endeavor that makes informed use of evidencecomprised of only public health records 10. A nurse who is working in a community setting applies evidence-based practice when planning community health education. Which of the following should the nurse use in order to enrich the application of this evidence? Randomized clinical trials Qualitative studies Quantitative studies Multiple research methods Otros 1. A community has residents who provide information to the city council so that decisions can be made about the health of the community. Which of the following characteristics is being displayed? Active partnerships Passive partnerships Gatekeeping Focal groups 2. A community health nurse is conducting informant interviews in a small community. Which of the following would the nurse most likely contact? The state department of health for death records Surrounding communities for crime comparison A local priest for congregation information The Centers for Disease Control and Prevention (CDC) for illnesses in the area 3. A nurse has collected data about the services available in the community and is using the seven “A” s to evaluate these services. Which of the following questions will the nurse most likely ask? Can the community members allocate resources to support the service? Can the community members afford the service? Will the community members allow strategies to be implemented to improve the service? Will the community members approve of the services provided? 4. A nurse has identified the boundaries of the community. Which of the following steps should the nurse take next when completing a community assessment? Plan interventions that benefit the entire community Formulate nursing diagnoses Gather relevant existing data and generating missing dataEvaluate the interventions that were used 5. A nurse interviews the school nurses in a community to determine their roles in schools because this data is not available. Which of the following processes is the nurse using? Primary data Secondary data Spatial data Photovoice 6. A nurse is caring for the community as the client. Which of the following is most likely the focus of the nurse’s care? Providing care for aggregates living in the community The collective good of the population The provision of care for families in the home setting Providing health education in the community 7. A nurse is completing a community assessment. Which of the following actions would be most likely for the nurse to complete? Determine the weaknesses of a community Assess individual needs within a community Perform the core functions of public health nursing Identify community needs and clarify problems 8. A nurse schedules an appointment with a physician who has a practice in the community to learn more about the community’s beliefs regarding childhood immunizations. Which of the following is being demonstrated? Participant observation Informant interview Active participation Windshield survey 9. What are the critical attributes in the definition of community? Families, groups, and health organizations Health needs, geographical boundaries, and target population People, place, and functions Populations and health resources 10. Which data source provides information about the function of the community? Local restaurant Elected officialsCounty health department Civic groups Otro 1. A community health center conducts blood lead screenings on a regular basis for children under 6 years old. Which of the following best describes the level of prevention that is being used? Tertiary prevention Primary prevention Secondary prevention Health education 2. Which one of the following services are not offered by Nurse-Led Medical Centers? immunization home visits critical care rehabilitation care 3. A nurse interviews the school nurses in a community to determine their roles in schools because this data is not available. Which of the following processes is the nurse using? Primary data Secondary data Spatial data Photovoice 4. A nurse is completing a community assessment. Which of the following actions would be most likely for the nurse to complete? Determine the weaknesses of a community Assess individual needs within a community Perform the core functions of public health nursing Identify community needs and clarify problems 5. Which levels of intervention are nurse-led centers designed to provide? Local, state, and national needs Individual, family, community needs Primary, secondary, and tertiary prevention Uninsured, under-insured, and insured populations 6. Which data source provides information about the function of the community?Local restaurant Elected officials County health department Civic groups 7. A nurse is caring for the community as the client. Which of the following is most likely the focus of the nurse’s care? Providing care for aggregates living in the community The collective good of the population The provision of care for families in the home setting Providing health education in the community 8. What are the critical attributes in the definition of community? Families, groups, and health organizations Health needs, geographical boundaries, and target population People, place, and functions Populations and health resources 9. A nurse schedules an appointment with a physician who has a practice in the community to learn more about the community’s beliefs regarding childhood immunizations. Which of the following is being demonstrated? Participant observation Informant interview Active participation Windshield survey 10. Which levels of intervention are nurse-led centers designed to provide? Local, state, and national needs Individual, family, community needs Primary, secondary, and tertiary prevention Uninsured, under-insured, and insured populations OTRO 1. A nurse is participating in the preparedness stage of disaster management. Which of the following describes what is happening? Heightened inspection and increased security in the community Incorporation of provision of pets into local disaster plans Purchase of personal protective equipment for all citizensAssembly of disaster kits for the home, workplace, and car 2. A nurse is working in a community that is experiencing the Honeymoon Phase after a disaster. Which of the following is characteristic of this phase? First responders work tirelessly to save others. Survivors share their stories. Medical personnel experience exhaustion. Community organizations rebuild the community 3. Acts of bioterrorism or natural disasters, such as earthquakes, will often have more casualties because: they cause the most widespread destruction victims have little time to make evacuation preparations those with chronic conditions cannot escape in time the early warning systems are not effective 4. What is the purpose of the National Incident Management System (NIMS)? Create a new branch of government that deals with bioterrorism Establish a way for the Red Cross to carry out its mission Extend presidential power to act quickly upon acts of bioterrorism Develop a nationwide all-hazards approach to domestic incident management 5. A riverfront community builds a retaining wall to divert flood water away from the town. Which term describes what has happened? Community resilience Rapid needs assessment Mitigation A natural disaster 6. A nurse responds to a natural disaster. Which of the following is the nurse most likely responding to? Transportation accident Pollution Communicable disease epidemic Fire 7. Which statement about disasters is true? They can be natural or human-made They can be relieved without assistanceThere is always injury and death when a disaster occurs The timing of a disaster does not influence the types of injuries that will occur 8. A nurse is working in a temporary shelter for victims following a natural disaster. Which condition is the nurse most likely to encounter? Stress Communicable disease Educational conferences Injuries requiring hospitalization 9. A nurse is assessing persons arriving at an alternate care center following a disaster. Which of the following actions should the nurse take first? Assess the amount of equipment and medications needed for each client Determine if the client has a psychological condition requiring special attention Assess whether this type of facility is appropriate for the client Determine if the client has a support system to assist with additional care needs 10. A nurse is working with an older person following a disaster. Which of the following emotions of the client should be anticipated by the nurse? Anger Fear of loss of independence Violence Happiness Community Final quizlet 136Q a school nurse is discussing levels of prevention with a teacher. which of the following activities should the nurse identify as a primary prevention strategy? a. provide nutritional counseling for students who have diabetes b. conduct vision and hearing screening for kindergarten enrollment c. demonstrate proper hand washing techniques d. report suspected child neglect to the proper authorities a school nurse is conducting hearing screening procedures in an elementary school. which of the following instructions should the nurse provide when preforming the Rinne test? a. after i place the tuning fork on your scalp, tell me if you hear the sound better in one ear or the same in both ears b. you'll wear headphones and press the button when you hear a sound c. after i place this tuning fork behind your ear, tell me when you no longer hear the soundd. use your finger to close one ear while i whisper some numbers into your other ear a school nurse is discussing levels of prevention with a teacher. which of the following activities should the nurse identify as a primary prevention strategy? a. provide nutritional counseling for students who have diabetes b. conduct vision and hearing screening for kindergarten enrollment c. demonstrate proper hand washing techniques d. report suspected child neglect to the proper authorities a school nurse is conducting hearing screening procedures in an elementary school. which of the following instructions should the nurse provide when preforming the Rinne test? a. after i place the tuning fork on your scalp, tell me if you hear the sound better in one ear or the same in both ears b. you'll wear headphones and press the button when you hear a sound c. after i place this tuning fork behind your ear, tell me when you no longer hear the sound d. use your finger to close one ear while i whisper some numbers into your other ear a palliative care nurse is teaching a client who has cancer about the services that are available for the client. which of the following statements should the nurse identify as an indication that the client understands the teaching? a. I can begin palliative care when i have less than 6 months to live b. this type of care can help me with pain control c. my family will not be involved with this type of care program d. if I begin palliative care, i will have to stop my chemotherapy a nurse is assessing a new client. which of the following information should the nurse include in the cultural portion of the assessment? a. employment status b. history of illness c. sexual orientation d. food preferences a nurse is collecting demographic data as a part of a community assessment. which of the following info should the nurse include? a. presence of condemned buildings b. racial distributionc. family genograms d. number of open water sources a nurse is caring for a client who has AIDS and is experiencing rapid weight loss. which of the following actions should the nurse take first? a. recommend the client increase her daily calorie intake by 25% b. teach the client about findings that should be reported to the provider c. encourage the client to consume 1.2-2.0g/kg of protein daily d. examine the client's oral mucous membranes a home health nurse is caring for a client who has breast cancer. which of the following assessment findings should the nurse identify as an indication that the client is coping effectively? a. excessive sleeping b. makes eye contact c. inability to concentrate d. lack of interest in food a home health nurse is scheduling client visits for the day. which of the following clients should the nurse plan to visit first? a. a client who is 10 days postoperative following a mastectomy and needs to have surgical staples removed b. a client in a hip spica cast who reports pruritus under the cast c. a client who has diabetes mellitus and reports new erythema to the left foot d. a client who has COPD and needs a follow-up visit related to home oxygen therapy a community health nurse is assessing a group of clients for risk factors of abusive behaviors. which of the following findings should the nurse identify as a risk factor for developing violent behavior? a. a client lives in the same neighborhood where he spent his childhood years b. a client states he witnessed his mother physically mistreat his father c. a client states that her mother-in-law provides childcare while she is working d. a client participates in volunteer activities in the community a school nurse is teaching health promotion to a group of staff members who sit at a desk and use a computer for 8 hr at a time. which of the following information is the priority for the nurse to include? a. take a walk after workb. have your visual acuity assessed regularly c. adjust your chair so your elbows are at desk height d. point and flex your toes periodically a case manager is planning an educational program for a client who has DM. which of the following activities should the nurse include when using the psychomotor domain of learning? a. show the client a video about how to monitor blood glucose levels b. give the client a pamphlet about foot care c. observe the client's technique for drawing up insulin d. review the color diagram of the food pyramid with the client a nurse manager at a community health clinic is presenting an in-service for nurses about assessing clients for abuse. which of the following statements by a nurse indicates an understanding of the teaching? a. I should not document the name of the person the client accuses of the abuse in the client's medical record b. i should wait until I see signs of physical abuse before I help the client develop a safety plan c. once I determine that a client is not at risk for abuse, I do not need to ask about it at future visits d. i should refer a client for a rape kit exam if she reports sexual assault within the previous 24 hours a nurse is teaching a client who has a new diagnosis of hepatitis A how to prevent the spread of the virus. which of the following instructions should the nurse include? a. clean your bathroom fixtures with a chlorine bleach solution b. rinse your toothbrush with hydrogen peroxide after each use c. use condoms during sexual contact for 2 weeks d. use shared hand towels to dry your hands after washing a community health nurse is developing a plan of care for a client who is hispanic. which of the following actions should the nurse include in the plan? a. maintain direct eye contact when speaking with the client b. avoid using hand gestures when working with the client c. discourage the client from using a faith healer d. use therapeutic touch during conversationa nurse is conducting an educational session at an assisted living facility for a group of clients who have osteoporosis. which of the following statements should the nurse include in the teaching? a. decrease your dietary intake of folate b. increase your daily intake of leafy green vegetables c. it is important to avoid weight-bearing exercises d. it is important to spend 30 min each day exposed to sunlight a hospice nurse is teaching about expected grief reactions with the family of a client who has end-stage pancreatic cancer. which of the following information should then use include? a. you will feel a sense of hopelessness throughout the grieving process b. it is common to experience a persistent state of sadness while grieving c. disturbances in your self esteem is an expected grief reaction d. a component of healthy grieving is the ability to openly express your anger a nurse is preparing an educational program about influenza for a group of community health nurses. which of the following activities should the nurse include as an example for tertiary prevention? a. provide immunizations at long-term care facilities b. offer classes to elementary school teachers about hand washing c. provide info to occupational nurses about the reasons for employees to not come to work d. administer antiviral meds within 48 hr to client who have manifestations fo influenza an occupational health nurse is planning to use an interpreter during an educational session with a group of migrant workers who do not speak the same language as the nurse. which of the following actions should then use take? (select all that apply) a. evaluate the interpreter's approach to clients prior to the educational session b. instruct the interpreter to guide the nurse in providing info in a culturally sensitive manner c. ask the interpreter to add info she feels may be necessary d. encourage the interpreter to paraphrase the workers' questions and responses e. choose an interpreter who speaks the workers language and dialect a nurse in a rural health clinic is caring for a client who has heart failure. the client states "I'm not going to take any more heart medicine." which of the following responses should the nurse make? a. why did you decide to stop your heart medicine? b. don't you know what stopping your medicine will do to your heart?c. don't you think your doctor knows what's best for your heart? d. can you tell me more about your decision to stop your medicine? a community health nurse is planning an educational program for farmers about occupational health risks. which of the following risks should the nurse include? a. respiratory disorders b. hypertension c. sickle cell anemia d. diabetes mellitus a community health nurse is working to meet the health care needs of residents in a rural community. which of the following characteristics should the nurse identify as a barrier to health care resources for this population? a. unavailability of outreach services b. less autonomy in providing client care c. lack of cohesiveness among community members d. disinterest by members of population in providing input for community health programs a community health nurse is teaching a client who is overweight about steps to take to begin an exercise program. the nurse should identify that which of the following statements is an indication that the client understands the teaching? a. I will need to purchase exercise equipment before I can start" b. I will see my doctor before beginning an exercise program c. i should try to perform aerobic exercise for an hour a day, 5 days a week d. I should avoid participating in weight-lifting exercises a public health nurse is participating in a community planning committee for disaster preparedness for a local community that is at risk for hurricanes. which of the following information should the nurse contribute to the plan? a. how to activate the local medical facility's emergency-management plan b. a list of residents who have experienced a hurricane in the past and will not need evacuation c. the name of the individual who is necessary to implement the plan d. a list of areas within the community where residents speak english as a second language a nurse s providing blood pressure screenings for older adult clients at a local community center. which of the following should the nurse identify as increasing a client's risk for developing hypertension a. HDL level greater than 70 mg/dLb. african american race c. daily walking routine d. BMI of less than 30 a home health nurse is planning care for the day. which of the following clients should the nurse visit first? a. an older adult client who has a newly prescribed antihypertensive medication and needs a BP check b. an older adult client who was treated in the ED last night for a stage III pressure ulcer c. a school-age child who was treated in the ED last night with status asthmaticus d. a school-age child whose percutaneous endoscopic gastrostomy (PEG) tube needs changing a community health nurse is working in a mobile health care clinic. which of the following clients should the nurse assess first? a. a client who has a new onset of confusion and slurred speech b. a client who has a temperature of 37.8C (100.1 F) c. a client who has COPD and an oxygen saturation of 92% d. a client who requires removal of sutures from a laceration a nurse in a community center is preparing to administer a tuberculin skin test to multiple clients to screen for TB. which of the following actions should the nurse take? a. insert the needle at a 45 degree angle b. create a wheal that measures about 15 mm in diameter c. preparing the outer aspect of the upper arm for the injection d. inject 0.1 mL of purified protein derivative a public health nurse is monitoring medication compliance for a group of migrant workers who are being treated for TB. the nurse should use info from which of the following resources to assist with this process? a. migrant clinicians network (MCN) b. U.S. preventive services task force (USPSTF) c. agency for healthcare quality and research (AHRQ) d. centers for disease control and prevent a school nurse is conducting visual acuity testing for a school-age child. which of the following actions should the nurse take? a. progress to the next line once the child reads two symbols correctlyb. have the child stand 5 feet away from the snellen letter chart c. allow the child to keep her glasses on during the testing d. begin the test by instructing the child to use both eyes to read the chart a public health nurse is developing a visual health program by using a community-oriented approach. which of the following interventions should the nurse include? a. provide genetic counseling to the family of a newborn who has congenital cataracts b. develop a plan of care for a client who is newly diagnosed with glaucoma c. teach a client who has a vision loss about safety in the home environment d. consult the local school nurse to schedule yearly vision screenings for students a public health nurse is planning care for four clients. which of the following interventions should the nurse recognize as tertiary prevention? a. providing chemoprophylaxis for malaria to a client who is traveling to mosquito-infested countries b. administering antibiotics to a client who has AIDS and was diagnosed with pneumocystis jiroveci c. performing a serological screening for HIV for a client who is pregnant d. participating in partner notification for a client who has an STI a nurse in a county health department is caring for a client who states, "I've been drinking too much in the evenings since my mom died last year." which of the following responses should the nurse make? a. it sounds like you are probably an alcoholic b. i don't think your mom would have approved of your drinking c. don't you think your family is being affected by your drinking? d. can i give you some information about alcoholic anonymous? a public health nurse is developing a presentation for local day care providers about infectious childhood diseases. which of the following statements should the nurse include? a. "children who have fifth disease will exhibit bloody diarrhea" b. "rotavirus infections in children peak during the summer months" c. "antiviral medications shorten the duration of shigella infection" d. "respiratory syncytial virus is spread through contact with respiratory secretions from an infected person" a community health nurse suspects an outbreak of scabies in the local area. which of the following actions should the nurse take first?a. determine the incidence rate b. institute prophylactic treatment c. educate the community about disease transmission d. discuss treatment plans with clients' families a community health nurse is discussing the concept of epidemiology with a coworker. which of the following interventions should the nurse provide as an example of the use of the principles of epidemiology? a. providing a client with info about preventing STIs b. reviewing county health records for data on cases of chickenpox c. conducting screenings for dental caries in the local school system d. documenting medication compliance for clients who have TB a community health nurse is assigned to lead a county-level environmental task force. which of the following activities should the nurse direct the task force to complete first? a. review community-specific epidemiological data b. recommend updates to local environmental policies c. distribute environmental health education materials to community members d. create program goals that align with healthy people 2020 objectives a community health nurse is teaching a client who was newly diagnosed with active pulmonary tuberculosis about disease transmission. which of the following information should the nurse include? a. household members should be placed in respiratory isolation b. the client should have a repeat purified protein derivative test in 3 months c. the client should use disposable utensils during meals d. household members should take isoniazid for 6-12 months The daughter of the client who has had a stroke asks the nurse for additional resources. What is the nurse's best response? a. increase the illumination in the room b. administer citalopram when the client is agitated c. give the client brief hand massages d. place a warm cloth on the clients foreheada community health nurse has been contacted regarding a client diagnosis of influenza type A in an adult day care. which fo the following actions should the nurse take to assist in the prevention of an outbreak? a. administer antiviral medication to clients at the facility b. recommend that the day care center close for 2 weeks c. schedule immunizations for clients at the facility d. give immune globulin to clients at the facility who have early manifestations of influenza a nurse in a rural community is planning education for a young adult client who is a migrant farm worker. which fo the following actions should the nurse include? (select all that apply) a. recommend a dental health screening b. provide skin cancer information c. provide forms to apply for medicare d. provide environmental health information e. refer the client for a tuberculosis screening a public health nurse is working in a community that has a population of 24,096. there are 2,096 existing cases of heart disease within the population. the nurse can determine which of the following from this information a. attack rate b. mortality rate c. incidence proportion d. prevalence proportion a home health nurse is assessing an older adult client who is taking captopril to treat heart failure. which of the following findings should the nurse identify as an adverse effect of this medication? a. dry cough b. weight gain c. ataxia d. photophobia a nurse is developing an educational program about bioterrorism and smallpox. the nurse should include in the teaching that the smallpox rash is expected to first appear in which of the following locations? a. lower extremities b. posterior shoulders c. faced. abdomen a nurse is assessing the risks and benefits of meal delivery services for an older adult client who lives alone and has no transportation. which of the following ethical principles is the nurse demonstrating? a. distributive justice b. fidelity c. beneficence d. respect for autonomy a nurse is conducting a home visit with a female client who reports a history of intimate partner abuse. the nurse should identify that which of the following findings places the client at greatest risk for domestic violence? a. the client visits friends without her partner's knowledge b. the client states she is leaving her partner c. the client is at 13 weeks of gestation d. the client recently started a new job a case manager is delivering a discharge plan for a client who has a spinal cord injury and is in a rehabilitation facility. which of the following actions should the nurse take first? a. contact service providers to determine the availability of services offered b. determine the client's ability to perform self-care c. hold a care conference with the client and his family to discuss treatment options d. evaluate the client's satisfaction with the case managers services a nurse at a county health clinic is caring for a client who has recently assumed the role of primary caregiver for her mother. which of the following statements by the client indicates that she is experiencing role conflict? a. i feel overwhelmed with not having enough time for my mom as well as my children b. i hope my siblings will be able to visit and help care form y mom for a few days c. i don't think my partner likes having to help more with the household chores d. i am glad that my job is flexible, so i can accommodate to my moms needs a nurse is planning to teach a community group about the meningococcal vaccine. the nurse should identify that which of the following clients should receive the vaccine? a. an infant who has bronchiolitis b. an infant who is 4 weeks old c. a client who is traveling to northern Europed. a client who is moving into a college dormitory a public health nurse is providing information to a client who has alcohol use disorder and is asking about treatment. which of the following statements should the nurse identify as an indication that the client understands the information? a. i am not eligible for an outpatient program until i have completed an inpatient program first b. i can expect to get help with other aspects of my life while in treatment c. once i make it through detoxification, i will be free of my addiction d. i will not have to completely stop drinking alcohol if i go into an inpatient treatment program a nurse on the scene following a mass casualty explosion is triaging a client who has a large, open occipital wound and the following findings: respiratory rate 6/min, atonal pattern; capillary refill time 4.5 seconds; non responsive to painful stimuli. which of the following actions should the nurse take? a. apply a cervical spine collar and perform a focus neurological exam b. turn the client to left semi-fowler's position and begin assessing the next client c. place a firm pressure dressing on the occiput and open the airway d. request that the client be assessed immediately by the next available provider a community health nurse is planning a presentation for adults who have a family history of Alzheimer's disease. which of the following behaviors should the nurse include as an early manifestation of alzheimers disease? a. experiencing incontinence b. withdrawal from social activities c. forgetting the location of common objects d. neglecting personal hygiene a community health nurse identifies an increase in the occurrence of osteoporosis related fractures in women experiencing menopause. which of the following primary prevention strategies should the nurse implement? a. encourage the women to participate in weight-bearing activities b. advise the women to keep their immunization updated c. educate the women about the importance of limiting sun exposure d. instruct at-risk women to increase there intake fo foods high in vitamin E a nurse is caring for a client who has terminal lung cancer and is receiving hospice care. which of the following statements should the nurse identify as an indication that the client is in the denial stage of the grief process? a. I'm going to plan my memorial service next weekb. I don't deserve to die. this just isn't fair c. If I could just make it through this, I'd never smoke again d. I'm looking forward to my daughter's wedding next year a home care nurse is visiting an older adult client and notes that unwashed dishes are piled up and newspapers cover the front steps. which of the following questions should the nurse ask the client to determine if he is socially isolated? a. have you considered moving to an assisted living facility? b. do you need help completing you're housework? c. how often do you have visitors come to see you? d. why haven't you brought in your newspapers? a community health nurse is discussing the role of a faith community nurse with a chaplain. which of the following information should the nurse include in the discussion? a. the faith community nurse can facilitate substance abuse support groups b. the faith community nurse can provide pharmacological pain management for clients who have a terminal illness c. the faith community nurse can plan workplace safety training for employees in a local factory d. the faith community nurse can provide wound care for clients in their home a community health nurse is conducting a needs assessment of a community. the nurse should identify that which of the following methods will yield direct data? a. health surveys b. morbidity/mortality statistics c. medical records d. informant interviews a nurse is providing education regarding lead exposure to a group of clients who live in a housing development built in 1968. which of the following client statements indicates an understanding of the teaching? a. i will increase the amount of red meat and milk in my child's diet b. i will use hot tap water to prepare my baby's formula c. i will vacuum our wood floors every week d. i will use a dry-sanding technique when preparing to repair my front door a community health nurse is participating in a quality improvement plan for a local health department. which of the following techniques should the nurse use for process evaluation of the facility? (select all that apply)a. written audits b. interviews c. values self-study d. satisfaction survey e. focus groups a community health nurse is planning an in-service about STIs for a group of adolescents. which of the following clinical findings should the nurse include as a manifestation of primary syphilis? a. lymphadenopathy b. maculopapular rash on palms c. chancre d. malaise a community health nurse is caring for an adolescent who is seeking help for an unplanned pregnancy. which of the following actions should the nurse take first? a. provide information on parenting classes so the client can learn about caring for a newborn b. help the client obtain a provider for prenatal care c. request permission to interview the father of the child to obtain a medical history d. recommend that the adolescent meet with the school guidance counselor to discuss educational options a community health nurse is planning to establish a community garden to address the lack of nutritious food options in the area. which of the following actions should the nurse take first to initiate the plan? a. monitor the progress of the project to keep the project on course b. select residents to take on leadership roles in the project c. identify community members who demonstrate an interest in the project d. hold a community information session to inform the residents of the plan a nurse is conducting a home visit for an older adult client. the nurse should identify which of the following findings as an indicator of possible neglect? a. lives alone b. taking outdated prescriptions c. has a BMI of 25 d. presence of alcohol in the homea nurse is caring for a 16 year old client who has a new diagnosis of human papilloma virus. which of the following actions should the nurse take? a. instruct the client to return for a blood test in 1 month b. report the infection to the state health department c. teach the client how to apply imiquimod 5% cream to the lesions d. administer ceftriaxone 250 mg IM a school nurse is reviewing the records of four students who are returning to school after being diagnosed with methicillin-resistant staphylococcus aureus (MRSA). which of the following actions should the nurse take? a. recommend prophylactic treatment of classmates b. provide education about MRSA throughout the school system c. report the cases of MRSA to child protective services d. coordinate an immunization clinic at the school a public health nurse is conducting an educational session about lyme disease for a group of older adult clients at a senior center. which of the following statements should the nurse identify as an indication that the client understands the teaching? a. i can get lyme disease from a mosquito bite b. i will have abdominal pain and diarrhea if i get lyme disease c. i should get an annual immunization to prevent lyme disease. d. i can take penicillin for 10-14 days to manage lyme disease a school nurse is educating a group of high school students about recommended dietary guidelines. which of the following statements by a student indicates an understanding of the teaching? a. i should consume less than 300 mg per day of dietary cholesterol b. i should consume 800 mg per day of dietary calcium c. i can increase my daily consumption of foods that contain refined grains d. i can consume up to 25% of my daily calories from saturated fatty acids a community health nurse is creating a program to reduce domestic violence in the community. which of the following interventions should the nurse identify as secondary prevention? a. collaborating with support agencies to ensure the ongoing treatment for abuse b. recognizing and reporting suspected abuse to the appropriate protective services c. educating individuals and groups about preventing domestic and community abuse d. creating a public service announcement about the warning signs of intimate partner abusea nurse in a clinic is planning teaching for a client who is newly diagnosed with hepatitis C. which of the following instructions should the nurse include in the teaching? a. schedule an appointment for an immunoglobulin injection b. wear a mask in public places while receiving treatment c. abstain from sexual intercourse until antibody tests are negative d. consume a low-carbohydrate diet until symptoms resolve a nurse at a country health department is caring for a client who is at 28 weeks of gestation. the nurse should identify which of the following characteristics as a risk for potential future abuse of the newborn? a. the client recently married the father of her unborn baby b. the client has changed providers three times during her pregnancy c. the client works part-time at a local resturant d. the client has recurring nightmares about her unborn baby a public health nurse is planning and educational program for a group of nurses at a community health department about pertussis infection. which of the following information should the nurse include? a. individuals who have had pertussis do not require immunization b. newborns should receive the first dose of the diphtheria, tetanus, and acellular pertussis (DTaP) vaccine prior to discharge from the hospital c. individuals should receive an annual influenza vaccine to minimize the risk fo infection with pertussis d. individuals transmit the pertussis bacteria through airborne droplets a clinic nurse is caring for a client who reports taking ginkgo biloba for several weeks since seeing a naturopathic healer. the nurse should instruct the client that ginkgo biloba may alter the effects of which fo the following medications? a. diltiazem b. warfarin c. digoxin d. metoprolol a faith community nurse is teaching the daughter of a client who has a terminal illness about her role as a member of the client's health care team. which of the following statements by the daughter indicates an understanding of the teaching? a. you will be able to access my mother's hospital medical records for us to review b. you can submit invoices to medicare to reimburse you for your servicesc. you will be able to give my mother pain medication d. you will coordinate with volunteers who will come to help my mother a home health care nurse is teaching a client's family about preventing the transmission of Clostridium difficile. which of the following transmission-based precaution should the nurse include in the teaching? a. protective environment b. airborne precautions c. droplet precautions d. contact precautions a nurse is performing a home visit for a client who has TB. as the nurse is leaving the client's house, a neighbor asks, "is it true that my neighbor has TB?" which of the following responses should the nurse make? a. you should take precautions against this infection b. have you ever been tested for TB? c. you should ask the public health department d. do you have questions about TB a nurse is developing a venogram for a client to determine education needs. which of the following health risks information should the nurse expect to obtain with this tool? a. economic b. social c. behavioral d. biological an occupational health nurse is assessing a client who reports taking ibuprofen daily. the nurse should counsel the client about the risk for which of the following adverse effects? a. hyperglycemia b. gastric ulcerations c. urinary retention d. orthostatic hypotension a nurse is giving a presentation about family violence at a local community center. which of the following information should the nurse include? a. child abuse is more common in homes where intimate partner abuse is present b. child who are abused are less likely to become abusersc. minority populations may be at greater risk for abuse d. intimate partner abuse occurs more frequently in lower socioeconomic households a nurse in an emergency department is triaging clients following an explosion at a local factory. which of the following clients should the nurse identify as a priority? a. a client who has agonal respirations and an open head injury b. a client who has tracheal deviation and shortness of breath c. a client who has a fracture of the humerus and a bleeding foot laceration d. a client who has superficial burns to 10% of the abdomen a public health nurse is planning a community health promotion program for hypertension prevention. which of the following interventions should the nurse include as a tertiary prevention strategy? a. implement an exercise program for clients who have hypertension b. teach clients who have family history of hypertension how to monitor blood pressure c. conduct a hypertension screening clinic for the community d. provide education about risk factors for hypertension a home health nurse is conducting a follow-up visit for a client who was recently discharged from an acute rehabilitation program for alcohol use disorder. which of the following actions should the nurse take? a. schedule transcranial magnetic stimulation (TMS) biweekly b. tell the client to take naltrexone daily c. teach the client to avoid foods that contain tyramine d. instruct the client to take buprenophine for the next 9 to 12 months a nurse is assessing a new client at a public health clinic which of the following areas should the nurse address as part of the cultural assessment? a. food allergies b. illness practices c. sexual activity d. immunization status a school nurse is notified that a school-age child has been newly diagnosed with pertussis. which of the following actions should the nurse take? (select all that apply) a. instruct the parent to keep the client at home until the coughing stage has passed b. encourage the family members to obtain prophylactic treatmentc. check the immunization status of the child's classmates d. recommend that the child receive a pneumococcal vaccine in 28 days e. quarantine the children in the child's class a nurse is caring for a client who has stage IV pancreatic cancer and has received information regarding available treatment options. which of the following is the responsibility of the nurse if the client chooses to forgo treatment and enter hospice care? a. assess whether or not the family agree with the clients decision b. verify that the clients health insurance pays for hospice services c. make the hospice referral in accordance with the clients decision d. recommend a second opinion for another provider a nurse is preparing a community education program about health care needs during pregnancy. the nurse should include that which of the following vaccines is safe to administer to client who is pregnant? a. tetanus, diphtheria, pertussis (TDaP) b. measles, mumps, rubella c. varicella d. herpes zoster a community health nurse is providing care to a client who has stopped taking his prescribed blood pressure medication. which of the following actions should the nurse take first? a. inform the provider of the client's decision b. provide the client with an educational pamphlet about the medication c. determine the client's reason for discontinuing the medication d. discuss the consequences of discontinuing the medication with the client community leaders have requested a meeting with a community health nurse to discuss creating a mobile meals program. which of the following should the community health nurse assess first? a. the need for the program b. the availability of volunteers c. the leadership support of the community d. the accessibility of residences a nurse is preparing to administer medication to a client who has active tuberculosis. which of the following precautionary measures should the nurse take? a. use an N95 respiratoryb. wear gloves c. use disposable equipment d. wear a gown a community health nurse is reviewing plans for a health education program. the nurse should identify that which of the following components of the plan needs to be changed? a. the presentation is delivered via a computer slide presentation b. pamphlets are written at a 12th grade level c. program content is organized topically d. attendance at the program is voluntary a nurse is planning health promotion activities for the local community. which of the following activities should the nurse include as an example of primary prevention? a. teaching foot care to adults who have diabetes mellitus b. providing tuberculosis screenings for day care providers c. testing school-age children for lead exposure d. teaching meal planning classes to older adults a home health nurse is providing nutritional instructions to a client who has COPD and is malnourished. which of the following instructions should the nurse include? a. avoid foods that contain eggs b. lie flat for 15 to 30 min after eating c. use milk instead of water when making canned soup d. drink at least 480 mL (16 oz) of liquid with each meal a nurse in a pediatric clinic is providing care to several clients. the nurse should recognize that which of the following conditions is included in the Nationally Notifiable Infectious Conditions list? a. varicella b. scarlet fever c. erythema infectious d. molluscum contagiosa a nurse is caring for a client who has recently emigrated from another country and states, "the health care system in my country was better and should be used everywhere." the nurse should recognize that the client is demonstrating which of the following behaviors? a. ethnocentrismb. social organization c. stereotyping d. cultural imposition a community health nurse is providing teaching about health promotion to a group of adolescents. which of the following topics is the most important for the nurse to include in an attempt to lower adolescent mortality rates? a. safer sex practices b. heart-healthy diet c. underage smoking d. safety belt use A nurse in an emergency department is planning care for a client who has a flail chest on the right side following a motor vehicle crash which of the following actions should the nurse plan to take? a. ask the client why he did not seek shelter sooner b. initiate a referral to the facility's social worker c. notify the local law enforcement agency of the client's situation d. tell the client everything will work out now that he is in the hospital a nurse in a community health clinic is preparing to administer an immunization to a 5 year old child. which fo the following actions should the nurse take? a. request that the child count backwards from the number 10 during the injection b. ask the child to pretend to blow up a balloon during the injection c. reassure the child that the injection is not going to hurt d. ask the child's parent to leave the room during the injection a nurse is developing a community education program about risk factors for family violence. the nurse should include which of the following circumstances as a risk factor for intimate partner abuse? a. attempting to end the relationship b. lacking supportive friends outside of the relationship c. having health issues that limit independence d. taking anti anxiety or sedative medications a home health nurse is planning care for a client who reports practicing traditional Asian health beliefs. the nurse should recognize that the client participates in which of the following actions? a. visits a shaman to seek healing from illnessb. applies cool compresses across the body to reduce fever c. places a cup of steam against the skin to draw out toxins from the body d. avoids eating dairy and meat products during the same meal a community health nurse is conducting vision screenings at a health fair for an older adult client who has age-related macular degeneration . which of the following statements should the nurse identify as an indication that the client is adapting to the changes? a. i have learned that i cannot go outside when the sun is bright b. i canceled all my magazine prescriptions since I can't read them c. i purchased green towels to use in my bathroom d. i have a prescription bottle magnifier to help me read my pill bottle labels a community health nurse is participating in a group session for clients who have alcohol and substance use disorders. which of the following information should the nurse provide regarding support programs for these individuals? a. alcoholics anonymous is a support group that requires disclosure of attendance to employers b. AA assists a client who has an addiction to alcohol with developing a daily recovery program c. narcotics anonymous will cure a client from her substance use disorder if she stays involved with the program d. NA is one-on-one program that assists clients an occupational health nurse in a factory is performing a routine tuberculosis screening and identifies an employee who has a positive mantis tuberculin test. which of the following actions should the nurse take? a. instruct the employee to prepare a list of close personal contacts b. initiate an employee immunization program c. instruct the employee to wear an N95 respiratory mask d. administer prophylactic penicillin to another employees A nurse manager at a community agency is developing an orientation program for newly hired nurses. When discussing the differences between community-based and community-oriented nursing, the nurse should include which of the following as examples of community-based nursing? (Select all that apply.) A. A home health nurse performing wound care for a client who is immobile B. An occupational health nurse providing classes on body mechanics at a local industrial plant C. A school nurse teaching a student who has asthma about medications D. A parish nurse teaching a class on low-sodium cooking techniquesE. A mental health nurse discussing stress management techniques with a support group A nurse is advocating for local leaders to place a newly approved community health clinic in an area of the city that has fewer resources than other areas. The nurse is advocating for the leaders to uphold which of the following ethical principles? A. Distributive justice B. Fidelity C. Respect for autonomy D. Veracity a nurse is preparing an education program on disease transmission for employees at a local day care facility. when discussing the epidemiological triangle, the nurse should include which of the following factors as agents? (select all that apply) a. resource availability b. ethnicity c. toxins d. bacteria e. altered immunity A nurse is developing a community health education program for a group of clients who have a new diagnosis of diabetes mellitus. which of the following learning strategies should the nurse include for clients who are auditory learners? a. showing informational videos b. providing equipment to practice hands-on skills c. supplying outlines for note-taking d. facilitating small group discussions a community health nurse is implementing health programs with several populations in the local area. in which of the following situations is the nurse using primary prevention? a. performing a home safety check at a client's home b. teaching healthy nutrition to clients who have hypertension c. providing influenza immunizations to employees at a local preschool d. implementing a program to notify individuals exposed to a communicable disease a nurse is preparing an educational program on cultural perspectives in nursing. the nurse should include that which of the following factors are influenced by an individual's culture? (select all that apply) a. nutritional practicesb. family structure c. health care interactions d. biological variations e. views about illness a nurse is caring for a client who is from a different culture than himself. when beginning the cultural assessment, which of the following actions should the nurse take first? a. determine the client's perception fo his current health status b. gather data about the client's cultural belief c. determine how the client's culture can affect the effectiveness of nursing actions d. gather information about previous client interactions with the health care system a nurse is using the I PREPARE mnemonic to assess a client's potential environmental exposures. which of the following questions should the nurse ask when assessing for "A" in the mnemonic? a. "what do you like to do for fun?" b. "what year was your residence built?" c. "what jobs have you had in the past?" d. "what industries are near where you live?" a nurse is reviewing information about the local health department to prepare for an interview. which fo the following services should the nurse expect the local health department to provide? (select all that apply) a. managing the women, infants and children program b. providing education to achieve community health goals c. coordinating directives from state personnel d. reporting communicable diseases to the CDC e. licensing of registered nurses a nurse is conducting health screenings at a statewide health fair and identifies several clients who require referral to a provider. which of the following statements by a client indicates a barrier to accessing health care? a. "I don't drive, and my son is only available to take me places in the mornings" b. "I can't take off during the day and the local after-hours clinic is no longer in operation" c. "Only one doctor in my town is a designated provider by my health maintenance organization" d. "I would like to schedule an appointment with the local doctor in my town who speaks spanish and english"a nurse is preparing to conduct a windshield survey. which of the following data should the nurse collect as a component of this assessment? (select all that apply) a. ethnicity of community members b. individuals who hold power within the community c. natural community boundaries d. prevalence of disease e. presence of public protection a nurse is completing a needs assessment and beginning analysis of data. which of the following actions should the nurse take first? a. determine health patterns within collected data b. compile collected data into a database c. ensure data collection is complete d. identify health needs of the local community a nurse is planning a community health program. which of the following actions should the nurse include as part of the evaluation plan? a. determine availability of resources to initiate the plan b. gain approval for the program from local leaders c. establish a timeline for implementation of interventions d. compare program impact to similar programs a nurse is conducting a community assessment. which fo the following data collection methods is the nurse using when having direct conversations with individual members of the community? a. key informant interviews b. participant observation c. focus groups d. health surveys a nurse is collecting data to identify health needs in the local community. which of the following examples should the nurse identify as secondary data? (select all that apply) a. birth statistics b. previous health survey results c. windshield survey d. community forume. health records a nurse is talking to a client who asks for additional information about hospice. which of the following statements should the nurse make? a. "clients who require skilled nursing care at home qualify for hospice care" b. "one function of hospice is to provide teaching to clients about life-sustaining measures" c. "hospice assists clients to develop the skills needed to care for themselves independently" d. "a component of hospice care is to control the client's manifestations" a school nurse is scheduling visits with a physical therapist for a child who has cerebral palsy. in which of the following roles is the nurse functioning? a. direct caregiver b. consultant c. case manger d. counselor an occupational health nurse is consulting with senior management of a local industrial facility. when discussing work-related illness and injury, the nurse should include which of the following factors as physical agents? (select all that apply) a. noise b. age c. lighting d. viruses e. stress A newly hired occupational health nurse at an industrial facility is performing an initial workplace assessment. Which of the following should the nurse determine when conducting a work site survey? A. Work practices of employees B. Past exposure to specific agents C. Past jobs of individual employees D. Length of time working in current role a school nurse is planning health promotion and disease prevention activities for the upcoming school year. in which of the following situations is the nurse planning a secondary prevention strategy? a. placing posters with images of appropriate hand hygiene near restrooms b. routinely checking students for pediculosis throughout the school yearc. implementing age-appropriate injury prevention programs for each grade level d. working with a dietitian to determine carbohydrate counts for students who have diabetes mellitus a nurse at a community clinic is conducting a well-child visit with a preschool-age child. the nurse should identify which fo the following manifestations as a possible indication of child neglect? (select all that apply) a. underweight b. healing spiral fracture of the arm c. genital irritation d. burns on the palms of the hands e. poor hygiene a nurse is caring for a client who is experiencing alcohol withdrawal. which of the following findings should the nurse identify as a manifestations of withdrawal? a. decreased blood pressure b. diaphoresis c. pinpoint pupils d. bradycardia a community health nurse is developing an education program on substance use disorders for a group of adolescents. which of the following information should the nurse include when discussing nicotine and smoking? a. smoking is the fifth-most preventable cause of death in the US b. nicotine is a central nervous system depressant c. withdrawal effects from smoking are minimal d. tolerance to nicotine develops quickly a community health nurse is developing strategies to prevent or improve mental health issues in the local area. in which of the following situations is the nurse implementing a tertiary prevention strategy? a. providing support programs for new parents b. screening a client whose partner recently died for suicide risk c. teaching a client who has schizophrenia about medication interactions d. discussing stress reduction techniques with employees at an industrial sitea nurse at an urban community health agency is developing an education program for city leaders about homelessness. which of the following groups should the nurse include as the fastestgrowing segment of the homeless population? a. families with children b. adolescent runaways c. intimate partner abuse survivors d. older adults a nurse is preparing a community health program on communicable diseases. when discussing modes of transmission, the nurse should include which of the following illness as airborne? a. cholera b. malaria c. influenza d. salmonellosis a home health nurse is discussing portals of entry with a group of newly hired assistive personnel. which fo the following locations should then ruse include as a portal of entry? (select all that apply) a. respiratory secretions b. skin c. genitourinary tract d. saliva e. mucous membranes a newly hired public health nurse is familiarizing himself with the levels of disaster management. which of the following actions is a component of disaster prevention? a. outlining specific roles of community agencies b. identifying community vulnerabilities c. prioritizing care of individuals d. providing stress counseling a community health nurse is educating the public on the agents of bioterrorism. which of the following agents should the nurse include as category A biological agents? (select all that apply) a. hantavirus b. typhus c. plagued. tularemia e. botulism a community health nurse is determining available and needed supplies in the event of a bioterrorism attack. the nurse should be aware that community members exposed to anthrax will need access to which of the following medications? a. metronidazole b. ciprofloxacin c. zanamivir d. fluconazole a nurse is creating partnerships to address health needs within the community. the nurse should be aware that which of the following characteristics must exist for partnerships to be successful? (select all that apply) a. being a leading partner with decision-making authority. b. flexibility among partners when considering new ideas c. adherence of partners to ethical principles d. varying goals for the different partners e. willingness of partners to negotiate roles a nurse is reviewing the various roles of a community health nurse. which of the following actions is an example of a nurse functioning as a consultant ? a. advocating for federal funding of local health screening programs b. updating state officials about health needs of the local community c. facilitating discussion of a client's ongoing needs with an inter professional team d. performing health screenings for high blood pressure at a local health fair a case management nurse at an acute care facility is conducting an initial visit with a client to identify needs prior to discharge home. after developing a working relationship with the client, the nurse is engaging in the referral process. which of the following actions should the nurse take first? a. monitor the client's satisfaction with the referral b. provide the client information to referral agencies c. review available resources with the client d. identify referrals that the client needsa nurse developing a community health program is determining barriers to community resource referrals. which of the following factors should the nurse include as an example of a resource barrier? a. costs associated with services b. decreased motivation c. inadequate knowledge of resources d. lack of transportation a nurse is working with a client who has systemic lupus erythematous and recently lost her health insurance. which of the following actions should the nurse take in the implementation phase of the case management process? a. coordinating services to meet the clients needs b. comparing outcomes with original goals c. determining the clients' financial constraints d. clarifying roles of inter professional team members Community 86 Q ATI urse who is facilitating a support group in a community center noticies that one member of the group expresses anger repeatedly. which of the following strategies should the nurse use to facilitate the group process with this member? A. remind the group that everyone should have a chance to participate D. divide the group into pairs and give each pair a topic for discussion C. give the member extra time to compose her thought before expressing them D. focus more on the group members who have a positive outlook the nurse should ask them in private to uncover the source of her ongoing anger a community health nurse is providing education to a group of children who have asthma. which of the following statements indicates an understanding of the teaching? A. i shouldn't play sports because it will make me too tired B. i will stay indoors during cold weather C. i will use my rescue medication every day D. I shouldn't get the flu shot because it might make me sick cold air can be a trigger for asthma. this response indicates the student reconizes this risk a community health nurse is planning an in-service about STI to a group of adolescents. which of the following clinical findings should the nurse include as a manifestation of primary syphilis? A. malaise B. maculopapular rash on palms C. chancre D. lymphadenopathya public health nurse is working in a community that ha a population of 24, 096. There are 2,096 existing cases of heart disease within the populaiton. The nurse can determine which of the following from this information? A. mortality rate B. attack rate C. prevalence proportion D. incidence proportion a nurse is preparing a community education program about heatlh care needs during pregnancy. The nurse should include that which of the following vaccines is safe to administer to a client who is pregnant? A. herpes zoster B. tetanus, diphtheria, pertussis C. varicella D. measles, mumps, rubella a school nurse is notified that an elementary school child is newly diagnosed with pertussis. Which of the following actions should the school nurse take? (select all that apply) A. instruct the parent to keep the child home until the coughing stage has passed B. encourage family members to obtain prophylactic treatment C. quarantine the children in the child's class D. recommend the child receive a pneumococcal vaccine in 28 days E. check the immunizations of the child's classmates a nurse is caring for a 50-year-old client who has DM, recently lost his job, and has no health insurance. The nurse should advise the client to do which of the following? A. contact medicare to determine eligibility B. contact medicaid to determine eligibility C. go to the ER when services are needed D. go to the public health department when services are needed low income, no insurance a home health nurse is orienting a newly licensed nurse who is drawing blood from a client for laboratory testing and gets stuck by a used needle. Which of the following statements by the newly licensed nurse indicates a need for further teaching? A. the client will be tested for HIV B. the client will be tested for hepatitis B virus C. the client will be tested for hepatitis C virus D. the client will be tested for hepatitis A virus D oral-fecal route a clinic nurse manager is developing an education program on TB skin testing. The nurse should include that which of the following can cause a false-negative result? A. advanced HIV or AIDS B. Bacilli CAlmette-Gue'rin (BCG) vaccination C. time between exposure and screening D. age of the client E. recent abuse of IV substancesa nurse is collecting demographic data as a part of a community assessment. which of the following should be included? A. racial distribution B. family genograms C. number of open water sources D. presence of condemned buildings a community health nurse is visiting a family that has newborn twims and two schoolaged children. The parents tell the nurse they are emotionally and financially overwhelmed. which of the following referrals are apporpriate? A. postpartum support group B. WIC program C. CPS D. medicare part B E. free school lunch program a community health nurse is teaching a group of older adults about environmental health hazards. A need for additional teaching is indicated when a group member lists which of the following as a possible source of carbon monoxide exposure? A. motor vehicles B. gas ranges C. vented gas heaters D. electric space heaters a community health nurse is caring for a client who has a new diagnosis of Lyme disease. the requirements for reporting Lymes disease are mandated by which of the following? A. local county health departments B. individual state law C. federal law D. centers for disease control an prevention a nurse is a community health center is talking with a client who is receiving treatment for an opioid addiction. which of the following statements by the nurse is apporpriate for encouraging the client to attend his 12 step meetings? A. you'll find Na will help you regain power over you addiction B. Na will help you take responsibility for the symptoms of your disease C. at Na, your willingness to change will be crucial for your sobriety D. at Na, you'll nearn to maintain control by never admitting to defeat a newly licensed nurse in a public health agency is assigned to assist with the task force that is responsible for the core function of asesment. which of the following is a task that the nurse could be assigned that would be consistent with the focus of the task force? A. investigating local health hazards B. updating local policies to support community health efforts C. collaborating with community agencies to improve availbility of services D. ensuring competence of health care workforce a home health nurse is assessing a client who has a prescription for captopril to treat heart failure. which of the following findings is an adverse effect of this medicaiton? A. weight gainB. ataxia C. photophobia D. dry cough a nurse is working for a non-profit organization with limited resources is asked to develop an education program that will benefit a supporting agency's employees. the nurse was planning another program to benefit more people in the community. which of the following is an ethical tenet the nurse should ofllowin in deciding which program to implement? A. allow members of the community to voice their shared ideas B. develop public policy in concert with the interests and needs of the community C. gibe precedence to community support provided by public servants over entrepreneurs D. place the needs of the community above those of special interest groups a nurse is developing an educational program about intestinal parasitic disease for a child day care staff. which of the following should the nurse emphasize to prevent transmission? A. spray snack tables with a baking soda solution weekly B. store children's personal items in separate lockers C. clean food preparation areas with soap and water daily D. wipe changing tables with a dilute bleach solution between children a nurse is providing immunizations for an infant in a clinic. which of the following statements made by the infant's parent indicates a need for further teaching? A. i can give my child acetaminiphen for fever after her immunizations B. i should reschedule my child's immunization if she has a cold C. my child will need an MMR vaccine at 12 months D. my child should not receive an influenza vaccine because she has an anaphulactic egg allergy a parish nurses's role include which of the following? A. providing pharmacologic pain management for clients with terminal illness B. planning safety services for employees in a clocal workplace C. performing wound care for clients in their homes D. facilitating substance abuse support groups a nurse has been granted funding to develop an education program that is specific to the local community. in what order shoujld the nurse perform the following steps? A. provide education within the context of environmental parameters B. develop objectives that must be achieved with the education C. select educational strategies that are apporpriate for the audience D. identify the health educatin needs of the community D B C A a community health nurse is discussing responsible behaviors with a client who has HPV. which of the following methods of transmission of HPV? A. oral contactB. droplet C. airborne D. sexual contact E. bloodborne a community health nurse suspects an outbreak of scabies which of the following actions should the nurse take first? A. educate the community about disease transmission B. determine the incidence rate C. institute prophylactic treatment D. discuss treatment plans with client's families a nurse is teaching a client who is newly diagnosed with active pulmonary tb about disease transmission. which of the following should the nurse include in the teaching? A. household members should be placed in respiratory isolation B. the client should use disposable utensils during meals C. household members should take isoniazid (INH) for 6-12 months D. The client should have a repeat purified protein derivative PPD test in 3 months a nurse at a local pediatric communiry clinic is prioritizing care for a group of clients. which of the following clients should the nurse see first? A. a child who is vomiting, febrile, and has developed a petechial rash B. an infant who has itching eyes with stringy discharge and red conjunctiva C. a child who has diarrhea, abdominal pain, and a fever of 37.4 (99.4) D. an infant who is irritable, pulling at her left ear, and has a fever of 38.2 (100.7) where does smallpox first develop? A. face B. posterior shoulders C. abdomen D. lower extremities a client started taking captopril 2 weeks ago. report what? A. angioedema B. bradycardia C. weight gain D. insomnia lifethreatening a community health nurse is reviewing plans for a health education program. which of the following components should be a cause for concern? A. program content is organized topicall B. pamphlets are written at a 12th grade level C. the presentation is delivered via powerpoint slides D. attendance at the program is voluntary should be 8th grade level a home health nurse is reviewing lab values from a group of clients. which of the following findings should be reported to the provider for evaluation? A. WBC 9000 B. T3 90C. HbA1c 5 D. PT 15 seconds a female who has MS is taking mitoxantrone IV every 3 months. What should be reported for adverse effects immediately to the provider? A. vomiting B. sore throat C. hair loss D. amenorrhea due to immunosuppression a nurse is giving a presentation on domestic abuse at a local community center. which of the following should hte nurse include in the teaching? A. minority populations may be at greater risk for abuse B. intimate partner abuse occurs more frequently in lower socioeconomic households C. child abuse is more common in homes where intimate partner abuse is present D. children who are abused are less likely to become abusers a hospice nurse is assessing a family's ability to care for a relative who has terminal cancer. which of the following findings indicates a potential barrier to the family's ability to provide necessary care? A. the family uses a wood stove as a primary source of heat B. the primary wage earner in the family recently became unemployed C. the family shares one vehicle among three adults D. members of the family have declined a mental health consult a public health nurse is providing education on how to prepare for an outbreak of avian influenza. which of the following should the nurse include in the teaching? A. individual's should receive the seasonal flu vaccine to minimize the risk of infection B. individual's should receive a single injection of the H5N1 vaccine C. individual's should take antiviral medication to prevent transmission D. individual's should have a 2 week supply of food and water at home a nurse is working in the ED when several hundred clients injured in a train collision arrive at the hospital for treatment. the traige nurse should determine that which of the following clients is in need of immediate treatment? A. a client who has neck pain and was transported to the hospital on a backboard B. a client who has epigastric and left arm pain and is diaphoretic C. a client who has nasal and orbital ecchymosis and epistaxis D. a client who has abdominal pain and is 2 months pregnant a community health nurse should know that, in the event of a smallpox bioterrorism threat, a mass inoculation plan would first target which of the following groups of people? A. newborns B. mortuary workers C. immunosuppressed clients D. clients who have eczema who cannot carry the smallpox virus? A. bodily fluids, such as blood or vomitB. contaminated objects, such as bedding and clothing C. bites from insects, such as mosquitoes D. inhalation of droplets, such as from coughing a nurse is working in a triage area. which of the following activities is unlikely to be the responsibility o the nurse in this setting? A. fostering positive public relations for the hospital B. performing a complete and comprehensive client assessment C. preventing cross contamination of infectious clients D. educating the client and family in the aftermath of a large earthquake, a hospital's disaster plan is put into effect. the charge nue must assemble a list of clients that can be safely discharge home to accommodate the anticipated admission of many victims. the charge nurse knows that it is unsafe to discharge which of the following clients? A. a client who has ostemyelitis and will require 6 weeks of IV antibiotics B. a client who has crohn's disease and is 1 day preoperative for an ileostomy C. a client who has AD and is awaiting placement in a long-term care facility D. a client who has an ileus following spinal surgery 5 days ago and is ambulatory in a brace pt will be NPO what is the presence of more than one disease or health condition in an individual at a given time? comorbidity a nurse is caring for employees and their family members who have become ill after a company picnic. after extensive interviews and a review of the food handling practices, the nurse determines that the most likely caue of the illness was a poultry dish that had been allowed to coo to room temperature for several hours before being served. The step in the epidemiological process is called which of the following? A. planning B. assessing C. implementing D. evaluating a public health nurse is visiting an older adult client who has chonic airflow limitation disease and is a former cigarette smoker. the client's medications include ipratopium (Atrovent) and albuterol (Proventil) inhalers, and the client has just been prescribed home oxygen to use as needed. The nurse should recognizethat the client's primary prevention needs include which of the following? A. periodic pulmonary function tests B. review of appropriate use of oxygen in the home C. yearly mammography examinations D. annual influenza immunizations a mother loses both of her children in a fire. upon learning about this the mother states "i can't believe they are gone, how will i make it through this?" what should the nurse respond with A. you are feeling overwhelmed right now B. don't worry, you will have plenty of helpC. can i call someone to be here with you D. anyone who has had a loss likethis would feel that way A a nurse is planning to participate in a public education program related to prevention of West Nile Virus. Which of the following instructions should the nurse include in the presentation? A. eliminate sources of standing water B. make sure immunizations are up to date C. keep all pets indoors D. spray nests with DEET A a family lives near a nuclear power plant and asks if there is anything they should havve on hand in case of a nuclear diaster, what would they be referring to? A. potassium iodide (Pima) B. potassium cyanide C. ciproflocxacin (Cipro) D. atropine (atropine sulfate) A if taken in time, can block the thyroid gland from the uptake of radioactive iodine and can reduce the risk of thyroid cancers a nurse is caring for a client who has a positive Mantoux skin test following screening for TB. The nurse should inform the client that the positive reaction indicates which of the following? A. the client has never been exposed to the terbercle bacillus B. the client had infectious TB at one time but it is not inactive C. the client has active TB D. further evaluation is required D a community health nurse should be aware that demographic modifying factors influencing healthy behaviors include which of the following? A. family health patterns B. income and educational levels C. self-esteem D. perceived health status B the living being that is affected by the agent host the animate or inanimate object that causes the disease agent the setting or surrounding that sustains the host environment the study of health-related trend in populations for the purposes of disease prevention, health maintenanc, and heath protection epidemiology the number of new cases in the populaiton at a specific time incidencethe number of existing cases in the population at a specific time prevalence what is the process of the epideiological process? put it in order A. make the plan B. using the gathered data, formulate a possible theory C. determine the nature, extent, and possible ignificance of the problem D. report and follow up E. evaluate the plan F put the plan into action G gather informaiton from a variety of sources in order to narrow down the possibilities C B G A F E D a nurse manager at a community agency is developing an orientation program for newly hired nurses. when discussing the differences between community-based and community-oriented nursing, the nurse should include which of the following situations as an example of community-based nursing? ( select all that apply.) a . a home health nurse performing wound care for a client who is immobile B. an occupational health nurse providing classes on body mechanics at a local industrial plant C. a school nurse teaching a student who has asthma about medications D. aparish nurse teaching a class on low-sodium cooking techniques . E. a mental health nurse discussing stress management techniques with a support group AC what does I PREPARE stand for in determining current and past environmental exposures? I-investigate potential exposures P- present work R-residence E-environmental concerns P-past workA-activities R-referrals E-educate medicare or medicaid? individuals must be older than 65 years old and receiving social security, have been receiving disability benefits for 2 years, have amyotrophic lateral sclerosis and receive disability benefits, or have kidney failure and be on maintenace dialysis or had a kidney transplant to qulify medicare medicare has how many parts? 4 what part of medicare? hospital care, home care, limited skilled nursing care part A what part of medicare? medical care, diagnostic services, physiotherapy part B what part me medicare? medicare advantage plan, a combination of A and B provided through private insurance part C what part of medicare? prescription drug coverage part D medicare or medicaid? provide health care coverage for individuals of low socioeconomic status and children, through the combined efforts of federal and state governments. eligibility is based on household size and income, with priority given to children, pregnant women, and those who have a disability medicaid medicare or medicaid? provides inpatient and outpatient hospital care, lab and radiology services, home health care, vaccines for children, family planning, pregnancy-related care, and early and periodic screening, diagnosis, and treatment services for those younger than 21 year old medicaid a nurse is preparing an educational progran on cultural perspectives in nursing. the nurse should include which of the following are influenced by an individual's culture? A. nutritional practices B. family structure C. healthcare interactions D. biological variations E. views about illnes A B C Ea nurse is caring for a client who is from a different culture than himself. when beginning the cultural assessement, which of the following actions should the nurse take first? A. determine the client's perception of his current health status B. gather data about the client's cultural beliefs C. determie how the client's culture can affect the effectiveness of nursing actions D. gather information about previous client interaction with the health care system B a nurse is conducting health screenings at a statewide health fair and identifies severe clients who require referral to a provider. which of the following statements by a client indicates a barrier to accessing health care? A. i don't drive, and my son is only available to take me places in the mornings B. i can't take off during the day and the local after-hours clinic is no longer in operation C. only one doctor in my town is a designated provider by my health maintenance organization D. i would like to schedule an appointment with the local doctor in my town who speaks spanish and english B a nurse is reviewing informaiton about the local health department to prepare for an interview. which of the following services should the nurse expect the local health department to provide? A. managing WIC B. providing education to achieve community health goals C. coordinating directives from state personnel D. reporting communicable disease to the CDC E. licensing of RNs B C what is the job of the state board of nursing? A. development and overight of the state's nurse practice act B. establishing public health policies C. licensure of RN and licensed practical nurses D. overight of the stats school of nursing E management of WIC B C D a nurse is preparing to conduct a windshield survey. which of the following data should the nurse ocllect as a component of this assessment? select all that apply A. ethnicity of community members B. individual's who hold power within the community C. natural community boundaries D. prevalence of disease E. presence of public protection A C Ea nurse is completing a needs assessment and beginning analysis of data. Which of the following actions should the nurse take first? A.Determine health patterns within collected data. B.Compile collected data into a database. C.ensure data collection is complete. D.identify health needs of the local community B A.nurse is planning a community health program. Which of the following actions should the nurse include as part of the evaluation plan? A. Determine availability of resources to initiate the plan. B. gain approval for the program from local leaders. C.establish a timeline for implementation of interventions. D.Compare program impact to similar programs D a nurse is conducting a community assessment. Which of the following data collection methods is the nurse using when having direct conversations with individual members of the community? A. Key informant interviews B.Participant observation C.Focus groups D.Health surveys A a nurse is collecting data to identify health needs in the local community. Which of the following examples should the nurse identify as secondary data? (Select all that apply.) A. Birth statistics B.Previous health survey results C.Windshield survey D.Community forum E. Health records A B E an occupational health nurse is consulting with senior management of a local industrial facility . when discussing work-related illnesses in injury, the nurse should include which of the following factors as physical agents? A. noise B. age C. lightingD. viruses E. stress A C a newly hired occupational healht nurse at an industrial facility is performing an initial workplace assessment. which of the following information should the nurse determine when conducting a work site survey? A. work practices of employees B. past exposure to specific agents C. past hjobs of individual employees D. length of time working in current role A a nurse at a community clinic is conducting a well-child visit with a preschool-age child. the nurse should identify which of the following a a manifestation of child neglect? A. underweight B. healing spiral fracture of the arm C. genital irritation D. burns on the palms of the hands E. poor hygiene A E a community health nurse is developing an education program on substance use disorders for a group of adolescents. which of the following should the nurse include when discussing nicotine and smoking? A. smoking is the fifth-most preventable cause of death in the US B. nicotine is a CNS depressant C. withdrawal effects from smoking are minimal D. tolerance to nicotine develops quickly D a nurse at an urban community health agency is developing an education program for city leaders about homelessness. which of the following should the nurse include as the fastest-growing segment of the homless population? A. families with children B. adolescent runaways C. intimate partner abuse victims D. older adults A a nurse is preparing a community health program on communicable diseases. when discussing modes of transmission, the nurse should include which of the following as an airborne illness? A. cholera B. malaria C. influenza D. salmonellosis Ca nurse is reviewing the various roles of a community health nurse. which of the following is an example of a nurse funcitoning as a consultant? A. advocating for federal funding of local health screening programs B. updating state official saobut health needs of the local community C. facilitating discussion of a client's ongoing needs with an interprofessional team D. performing health screenings for high blood pressure at a local health fair [Show More]

Last updated: 1 year ago

Preview 1 out of 99 pages

Reviews( 0 )

$19.00

Add to cart

Instant download

Can't find what you want? Try our AI powered Search

OR

GET ASSIGNMENT HELP
243
0

Document information


Connected school, study & course


About the document


Uploaded On

May 13, 2020

Number of pages

99

Written in

Seller


seller-icon
Kirsch

Member since 4 years

898 Documents Sold


Additional information

This document has been written for:

Uploaded

May 13, 2020

Downloads

 0

Views

 243

Document Keyword Tags

Recommended For You


$19.00
What is Browsegrades

In Browsegrades, a student can earn by offering help to other student. Students can help other students with materials by upploading their notes and earn money.

We are here to help

We're available through e-mail, Twitter, Facebook, and live chat.
 FAQ
 Questions? Leave a message!

Follow us on
 Twitter

Copyright © Browsegrades · High quality services·